Está en la página 1de 68

Universidad La Salle.

Facultad Mexicana de Medicina.


Curso de extensión universitaria para la Preparación del Examen Nacional para
Aspirantes a Residencias Médicas.
Examen Módulo Medicina Interna
12 MAYO 2010.
Modalidad a distancia.

Nombre: Examen Módulo III de Medicina Interna.


Duración: 2 horas.

1.-Se trata de femenino de 40 años de edad acude al servicio de urgencias quejándose de


calambres en las piernas y parestesias en los dedos de las manos. Un año antes se le
sometió a una operación del cuello, pero no está segura qué fue lo que se le hizo. El signo de
Chevostek es positivo: La percusión sobre el nervio facial por delante de la oreja
desencadena una contracción espasmódica del labio superior. ¿Cuál de los siguientes
trastornos sugiere un signo positivo?

a) Hipercalciemia.
b) Acidosis.
c) Hiperpotasemia.
d) Hipocalciemia.

SINTOMATOLOGÍA
Neuromuscular: La hipocalcemia aguda se manifiesta por parestesia (hormigueo y
adormecimiento de los dedos y región peribucal) y calambres o contracturas musculares. La
sintomatología subclínica de tetania, evidenciada por el signo de Chvostek, (ocurrencia de
espasmo facial, especialmente del orbicular de los labios, al percutir el nervio facial a mitad
de distancia entre la comisura labial y el oído). El signo de Trousseau es un espasmo
doloroso del carpo, que se presenta luego de mantener por tres minutos una presión >20
mmHg por encima de la sistólica, siendo un signo más de tetania.
La hipocalcemia crónica se presenta con irritabilidad, confusión, demencia e incluso, en
infantes, como retardo mental. También se reportan movimientos coreicos, distonías y
convulsiones. Se ha reportado calcificación de ganglios basales en la radiografía de cráneo,
que no es reversible al tratamiento.

Cardiovascular: Prolongación de la fase de potencial de acción y por lo tanto prolongación


del segmento ST en el ECG. En casos de severa deficiencia se presentan arritmias,
hipotensión o falla cardiaca; la hipocalcemia aumenta la cardiotoxicidad de los digitálicos.

Pulmonar: Broncoespasmos y laringoespasmos vistos, sin embargo, con poca frecuencia.

Dermatológica: Piel seca, uñas quebradizas y caída del cabello como signos no específicos
en la hipocalcemia crónica.
LECTURAS RECOMENDADAS:

1. Carlstedt F, Lind L. Hypocalcemic syndromes. Crit Care Clin 2001; 17:139-153.


2. Gibbs M, Wolfson A, Tayal V. Electrolyte disturbances. En: Rosen´s Emergency Medicine.
Concepts and Critical Practice. J Marx, Hockberg
R, Walls R, et al (eds). Fith edition. Mosby. St Louis,1998.
3. Kapoor M, Chan GZ. Fluid and electrolyte abnormalities. Crit Care Clin 2001; 17:503-529.
4. Lind L, Carlstedt F, Rastad J, et al. Hypocalcemia and parathyroid hormone secretion in
critically ill patients. Crit Care Med 2000; 28:93-99.
5. Lo CY. Postthyroidectomy hypocalcemia. J Am Coll Surg 2003; 196:497-498.
6. Marx SJ. Hyperparathyroid and hypoparathyroid disorders.N Engl J Med 2000;
343:1863-1875.
7. Pegoraro A, Rutecki G. Hypomed/topic1118.htm

2.- Masculino de 47 años de edad que cursa con un ataque agudo de gota, la articulación
que se afecta con mayor frecuencia en éste caso es:

b) La rodilla.
c) La muñeca.
a) La primera metatarso-falángica.
d) El codo.

El American College of Rheumatology tiene 11 criterios, y la presencia más o menos de seis


sugiere la presencia de gota. Los 11 criterios son:

1. Más de un ataque de artritis activa.


2. Inflamación máxima desarrollada a lo largo de un día.
3. Ataque de oligoartritis.
4. Enrojecimiento observado en la articulación.
5. Primera articulación metatarsofalángica dolorosa o inflamada.
6. Ataque unilateral de la primera articulación metatarsofalángica.
7. Ataque unilateral de la articulación del tarso.
8. Tofo (probado o sospechado).
9. Hiperuricemia.
10. Hinchazón asimétrica en la radiografía dentro de una articulación.
11. Terminación completa de un ataque.

Coll JM, Blanch J. Hiperuricemia y gota. Aspectos terapéuticos. Jano 1997;1201:47-


52.
González Barber A, Carlavilla AB. Guía en Hiperuricemia y Gota. Madrid:
EDIMSA, 2003.
Nadal A. Farmacovigilancia. Alopurinol: aumento en la incidencia de episodios agudos de
gota. Offarm 2000;19(1):128.
Pérez Ruiz F, Calabozo Raluy M, Ugalde Espiñeria J, Herrero Beites AM.
Artropatías microcristalinas I. Hiperuricemia y gota. Medicine
2001;8(34):1765-72.
Sánchez Pozo A, Faus MJ. Hiperuricemia y gota. Pharm Care Esp 2003;5:105-9.
Sancho Bueso T, Bernardino de la Serna I, García Puig J. Consulta diaria.
¿Qué haría usted ante un paciente con hiperuricemia? Medicina Integral 2000;3(35):100-11.
Sancho Bueso T, García Puig J. Criterios terapéuticos ante la hiperuricemia.
Revista Clínica Española 2001;2(201):85-7.

3.- Se reportaron 45 casos de hepatitis A entre los alumnos de tercer grado de una
escuela secundaria durante la segunda quincena del mes de agosto. La tasa de ataque es de
26.4%. Esta situación nos indica que estamos ante un(a):

a) Epidemia.
b) Endemia.
c) Brote.
d) Pandemia.

De acuerdo a la NOM- 017 un brote se define como la ocurrencia de dos o más casos
asociados epidemiológicamente (tiempo, lugar y persona) entre sí. La medida cuantitativa
de la extensión de un brote es la Tasa de Ataque (TA) que se calcula dividiendo el número
de casos nuevos entre el total de personas expuestas por 100.
Greenberg R. S; Epidemiología médica, Manual Moderno, 2ª. Ed. Págs. 77-79.

4.- Se realiza el diagnóstico de neumonía atípica, esta patología clásicamente es producida


por:

a) Micoplasma Pneumoniae.
b) S. pneumonia.
c) Proteus.
d) Salmonella.

La neumonía por micoplasma es un tipo de neumonía atípica y es causada por la bacteria M.


pneumoniae. Este tipo de neumonía generalmente afecta a personas menores de 40 años.
Diversos estudios sugieren que esta enfermedad comprende entre el 15 y el 50% de todos
los casos de neumonías en adultos e incluso más en los niños en edad escolar.
Los agentes atípicos tales como Mycoplasma pneumoniae y Chlamydia pneumoniae producen
generalmente neumonías de evolución más benigna; sin embargo, éstas pueden ser graves,
aún en personas previamente sanas. El diagnóstico serológico de estos microorganismos
tiene varias limitaciones: se requiere un par serológico, los puntos de corte para el título de
anticuerpos son variables en los estudios y las técnicas de medición de la IgM no están
siempre disponibles. Se ha estimado que alrededor del 1% a 10% de los casos de neumonía
comunitaria grave serían causados por estos microorganismos.
Chiner E, Signes-Costa J, Andreu AL, Andreu L. Neumonía por Mycoplasma pneumoniae: una
rara causa de distrés respiratorio del adulto. An Med Interna (Madrid) 2003; 20: 11.
Radisic M, Torn A, Gutiérrez P, Defranchi HA, Pardo P. Severe Acute Lung Injury caused
by Mycoplasma pneumoniae: Potential Role for Steroids Pulses in Treatment. Clin Infect
Dis 2000; 31: 1507-11.
Gleason PP. The emerging role of atypical pathogens in community-acquired pneumonia.
Pharmacotherapy 2002; 22 (1Pt 2): 2S-11S.
Pflouffe JF. Importance of atypical pathogens of community-adquired pneumonia. Clin
Infect Dis 2000; 31 (Supl. 2):S35-9.

5.- Paciente de mediana edad que llega al servicio de urgencias con dolor torácico de más
de 12 hrs. de evolución, que inicia en forma súbita, mejora al estar sentado, no tiene
antecedentes de importancia, habrá que descartar de primera instancia:

a) TEP.
b) Enfermedad ácido péptica.
c) Cardiopatía isquémica.
d) Lesión de grandes vasos.

El manejo de un paciente con este tipo de dolor, requiere una evaluación de la severidad,
localización y características peculiares de dicho dolor. Muy pocos síntomas suponen una
urgencia tan obligatoria como lo es el dolor torácico. Tanto el médico como el paciente
saben que la isquemia miocárdica puede ser causa de muerte súbita, pudiendo generar
ansiedad en ambos. La importancia y dificultad en la valoración del dolor torácico radica en
la multitud de causas posibles y en el diferente pronóstico según la patología subyacente.
Al problema diagnóstico inherente a un síndrome esencialmente clínico, se añade la
dificultad de etiquetar el dolor en poco tiempo (ayudados sólo por la clínica, una Rx de
tórax y un ECG), dada la importancia de iniciar con prontitud el tratamiento más adecuado
en los pacientes con patología potencialmente letal. Aunque el dolor o la molestia precordial
constituye una de las manifestaciones principales de cardiopatía, es muy importante
recordar que puede originarse no sólo en el corazón, sino también en: estructuras cardiacas
intratorácicas como la aorta, la arteria pulmonar, árbol broncopulmonar, pleura, mediastino,
esófago y diafragma; tejidos del cuello o la pared torácica, incluidos piel, músculos
torácicos, región cervicodorsal, uniones costocondrales, mamas, nervios sensoriales o

médula espinal y órganos abdominales como estómago, duodeno, páncreas o vesícula biliar;
además de dolor artificial o funcional.

Manifestaciones clínicas en cardiopatía isquémica

Estas se pueden agrupar en cuatro grandes grupos o categorías, las cuales son:

Angina de pecho: en donde la obstrucción del riego arterial no es lo suficientemente


persistente como para causar muerte del tejido muscular cardíaco; hay tres tipos que son
la angina estable, la de prinzmetal y la inestable.
Infarto del miocardio: en este caso la obstrucción del riego arterial es lo suficientemente
duradera o persistente como para causar necrosis tisular isquémica del miocardio.

Cardiopatía isquémica crónica: son pacientes que generalmente han sufrido uno o más
ataques cardíacos y han sobrevivido a ellos, pero continúan teniendo problemas cardíacos
debido a que la parte del miocardio que no muere se hipertrofia para suplir las necesidades
del cuerpo y esto a su vez causa un aumento de la demanda cardíaca debido al aumento de
los componentes estructurales de las células cardíacas, trayendo más problemas porque no
se podrá suplir adecuadamente al corazón debido a la obstrucción coronaria. Estos
pacientes constituyen el 50% de los que reciben trasplantes cardíacos.

Muerte súbita cardíaca: Es el paro cardíaco en el cual se presentaron síntomas en una


hora antes de la muerte, o no se presentaron nunca. Causas: aterosclerosis coronaria,
estenosis aórtica, hipertensión sistémica, comúnmente arritmias letales (asistólicas y
fibrilación ventricular).

BIBLIOGRAFIA
1. Goldman L., Braunwald E. Molestias torácicas y Palpitaciones. En Isselbacher KJ.,
Braunwald E., Wilson JD., Fauci AS., Kasper DL., eds. Harrison, Principios de
Medicina Interna. McGraw-Hill. Interamericana de España. 1994.
2. Braunwald E. Tratado de Cardiología. Interamericana. Mcgraw-Hill. 1993.
3. Harkins SW. Geriatric pain. Pain perceptions in the old age. Clin Geriatric Med
1996.
4. Coto lópez, A., Morales JM., Gutierrez Rodero, F., Gonzalez E., .Dolor Torácico.
Manual de diagnóstico y terapéutica médica. Gutierrez Rodero F y García Díaz JD.
2ª ed. Madrid, 1990; pag. 165-172.
5. Durán Serantes, M., Calderón de la Barca Gázquez, J.M., Romero Moreno M.,
Martinez Guillén, J., Montero Pérez, FJ., Jimenez Murillo, L., Cardiopatía
Isquémica ( I ): Angor. Protocolos de actuación en Medicina de Urgencias.
Jimenez Murillo L y Montero Pérez FJ. Mosby/Doyma Libros SA. Barcelona
1996; pág. 51-56.
6. James H. Chesebro. La clínica del dolor torácico en el Servicio de Urgencias: abordaje
de los pacientes y relación coste-eficacia. Grandes temas de la cardiología: avances hacia
el cambio de siglo. 1998, American College of Cardiology.
7. Tresch DD, Aronow Ws. Clinical manifestations and clinical diagnosis of coronary artery
disease. Clin Geriatr Med. 1996.
8. Owens, G.M.: Chest pain. Primary Care, 1986. 13; pág: 55-61.
9. Rutherford, J.D.; Braunwald, E.: Diagnóstico diferencial del dolor precordial. En:
Braunwald E: Tratado de Cardiología, 4ª edición. Interamericana McGraw-Hill,
Madrid, 1993; pág: 1448-1449.
10. Williams, E.S.: Approach to the patient with chest pain. En: Kelly WN, ed. Textbook of
Internal Medicine. Filadelfia, J.B. Lippincott Company, 1989; pág. 374-379.
6.- Acude a consulta un adolescente de 20 años la cual refiere que desde hace mas de un
año presenta astenia, cansancio, pérdida de apetito y dificultades para concentrarse en los
estudios. Al interrogatorio refiere que a perdido interés en los estudios, frecuenta menos
a sus amigos, con pesimismo en la mayoría de sus actividades. ¿Cuál es el diagnóstico más
probable?

a) Anorexia nerviosa.
b) Trastorno de ansiedad.
c) Trastorno Distímico.
d) Depresión mayor.

Criterios para el diagnóstico de


F34.1 Trastorno distímico (300.4)

A. Estado de ánimo crónicamente depresivo la mayor parte del día de la mayoría de los días,
manifestado por el sujeto u observado por los demás, durante al menos 2 años.

Nota: En los niños y adolescentes el estado de ánimo puede ser irritable y la duración debe
ser de al menos 1 año.

B. Presencia, mientras está deprimido, de dos (o más) de los siguientes síntomas:

1. pérdida o aumento de apetito.


2. Insomnio o hipersomnia.
3. Falta de energía o fatiga.
4. Baja autoestima.
5. Dificultades para concentrarse o para tomar decisiones.
6. Sentimientos de desesperanza.

C. Durante el período de 2 años (1 año en niños y adolescentes) de la alteración, el sujeto no


ha estado sin síntomas de los Criterios A y B durante más de 2 meses seguidos.

D. No ha habido ningún episodio depresivo mayor durante los primeros 2 años de la


alteración (1 año para niños y adolescentes); por ejemplo, la alteración no se explica mejor
por la presencia de un trastorno depresivo mayor crónico o un trastorno depresivo mayor,
en remisión parcial.

Nota: Antes de la aparición del episodio distímico pudo haber un episodio depresivo mayor
previo que ha remitido totalmente (ningún signo o síntoma significativos durante 2 meses).
Además, tras los primeros 2 años (1 año en niños y adolescentes) de trastorno distímico,
puede haber episodios de trastorno depresivo mayor superpuestos, en cuyo caso cabe
realizar ambos diagnósticos si se cumplen los criterios para un episodio depresivo mayor.

E. Nunca ha habido un episodio maníaco, un episodio mixto o un episodio hipomaníaco y


nunca se han cumplido los criterios para el trastorno ciclotímíco.
F. La alteración no aparece exclusivamente en el transcurso de un trastorno psicótico
crónico, como son la esquizofrenia o el trastorno delirante.

G. Los síntomas no son debidos a los efectos fisiológicos directos de una sustancia (p. ej.,
una droga, un medicamento) o a enfermedad médica (p. ej., hipotiroidismo).

H. Los síntomas causan un malestar clínicamente significativo o deterioro social, laboral o


de otras áreas importantes de la actividad del individuo.

Especificar si:
Inicio temprano: Si el inicio es antes de los 21 años.
Inicio tardío: Si el inicio se produce a los 21 años o con posterioridad.

Especificar (para los últimos 2 años del Trastorno distímico):

Con síntomas atípicos

DSM IV

MANUAL DIAGNÓSTICO Y ESTADÍSTICO DE LOS TRASTORNOS MENTALES


(American Psychiatric Association)

7.- ¿Estamos ante una epidemia cuando?

a) La tasa de ataque de la enfermedad sea mayor de 10 por 1000 habitantes.


b) Los casos de una enfermedad aumentan en exceso con respecto a la
expectativa normal para una población en ese momento.
c) Una enfermedad tiene una tasa baja de aparición pero se encuentra de manera
constante en una comunidad o región.
d) Los casos de una enfermedad aumentan en una determinada estación del año.

Enfermedad que por un tiempo afecta a un gran número de personas que viven en una misma
región.
Enfermedad que por alguna temporada afecta a un pueblo o comarca, padeciéndola
simultáneamente un gran número de personas.
Se denomina a la aparición brusca de una enfermedad que se disemina rápidamente entre
una población determinada. A veces se la identifica erróneamente como brote. La forma
más habitual es la producida por virus, debido a su hábito cambiante.

Boffi H, Álvarez-Herrera C. Contribución al estudio de las enfermedades


transmisibles mediante el uso de algunos métodos estadísticos. Salud Argentina
1970;1:13–97.
8.- En un paciente con insuficiencia suprarrenal primaria podemos encontrar las siguientes
alteraciones de laboratorio:

a) Hipercalcemia e hipernatremia.
b) Hipernatremia e hiperglucemia.
c) Hipocalcemia y leucopenia.
d) Hipercalemia e hipoglucemia.

En la insuficiencia suprarrenal primaria se observa disminución en los niveles séricos de


sodio, cloruro y bicarbonato, mientras que el potasio sérico se eleva. Esta se debe a los
efectos combinados de déficit de aldosterona, reducción del filtrado glomerular y acidosis.
Y sobre todo durante el estrés puede haber disminución en los niveles de glucosa.

• Datos de laboratorio:

1. Hiponatremia: Los niveles séricos bajos de sodio se debe a su pérdida por la orina
por déficit de aldosterona y al desplazamiento del sodio hacia el compartimento
intracelular. Esta pérdida de sodio extravascular reduce el volumen plasmático y
acentúa la hipotensión.
2. Hiperkalemia: Aumento de los niveles séricos de potasio. Se debe a los efectos
combinados del déficit de aldosterona, la reducción del filtrado glomerular y la
acidosis.
3. Hipocortisolemia: Los niveles de cortisol y aldosterona son bajos y no aumentan con
la administración de ACTH.
4. Hipercalcemia: Aumento de los niveles séricos de calcio. Ocurre en un 10-20% de
los pacientes de causa desconocida.
5. Cambios electrocardiográficos: Suelen ser inespecíficos, aunque con ldentificación
generalizada del trazado.
6. Hemograma: Puede haber anemia normocítica, linfocitosis relativa y eosinofilia
moderada.
7. Prueba de estimulación de ACTH: Prueba principal que confirma el diagnóstico de
insuficiencia suprarrenal, al evaluar la capacidad de las suprarrenales para producir
esteroides, que suelen estar ausentes o disminuidos tanto en sangre como en orina
tras la estimulación de ACTH.
8. Determinación de la ACTH: En la insuficiencia suprarrenal primaria o Enfermedad
de Addison, la ACTH y sus péptidos afines, están elevados en plasma ante la
pérdida del mecanismo de retroalimentación del eje hipotálamo-hipófisario-
suprarrenal.
9. Hipertermia: La hormona del hipotálamo no controla la homeostasis.

Williams GH, Dluhy RG. Enfermedades de la corteza suprarrenal. En Jameson JL (ed):


Harrison. Endocrinología. 1a ed. Madrid. MacGraw-Hill España, 2006: 137-138.
9.- Se presenta ante usted femenino de 34 años con referencia por diagnóstico de lupus
eritematoso generalizado, ¿Cuál es el síntoma dentro de los criterios de clasificación de
de éste padecimiento?

a) Alopecia.
b) Fotosensibilidad.
c) Artralgias.
d) Pérdida de peso.

Criterios de Clasificación para el Diagnóstico de Lupus Eritematoso Sistémico (LES)


Erupción malar: Eritema fijo, plano o alto, sobre las eminencias malares, que no suele
afectar los surcos nasogenianos.
Erupción discoide: Placas eritematosas altas, con descamación queratósica adherente y
tapones foliculares; puede haber cicatrices atróficas en las lesiones más antiguas.
Fotosensibilidad: Erupción cutánea a causa de una reacción insólita a la luz solar,
referida por el paciente u observada por el médico.
Úlceras bucales: Ulceración nasofaríngea, por lo común indolora, observada por un médico.
Artritis: Artritis no erosiva que afecta dos o más articulaciones periféricas, caracterizada
por dolor a la palpación, tumefacción o derrame. Serositis: Pleuritis o pericarditis
documentada por electrocardiograma o frote o evidencia de derrame pericárdico.
Enfermedad renal: Proteinuria persistente mayor a 0,5g/día o 3+ o cilindros celulares.
Trastorno neurológico: Convulsiones o psicosis en ausencia de otra causa conocida.
Trastorno hematológico: Anemia hemolítica o leucopenia (< 4.000/mm3) o linfopenia: (<
1.500/mm3) o trombocitopenia (< 100.000/mm3) en ausencia de fármacos que produzcan
esta alteración.
Trastorno inmunológico: Anti-DNA, anti-Sm, y/o Anticuerpos antifosofolipídicos (AFL).
Anticuerpo antinuclear: Un título anormal de ANA por inmunofluorescencia o análisis
equivalente en cualquier momento y en ausencia de medicamentos relacionados con el
síndrome de lupus de origen farmacológico.

Cualquier combinación de 4 o más de los 11 criterios, bien documentado durante cualquier


intervalo de la historia del paciente, hace el diagnósticos de LES (especificidad y
sensibilidad son del 95% y 75%, respectivamente).

Petri M. Review of classification criteria for systemic lupus erythematosus. RheumDis


Clin North Am. 2005 May;31(2):245-54.

10.- Cuando nos enfrentamos ante un cuadro clínico de amenorrea, galactorrea y pérdida
de campo visual el diagnóstico más probable es:

a) Adenoma hipofisario no funcionante.


b) Intoxicación por benzodiacepinas.
c) Prolactinoma.
d) Meningioma del tubérculo solar.
El prolactinoma es el tumor hipofisario más frecuente, es de naturaleza benigna y pequeño
en 90 % de los casos. El cuadro clínico típico en la mujer se compone de trastornos
menstruales, galactorrea y/o esterilidad; se acompaña de síntomas neurológicos sólo
cuando se extiende por arriba de la silla turca. Niveles de prolactina superiores a 100
ng/mL son prácticamente diagnósticos de prolactinoma, siempre y cuando no existan
embarazo y/o hipotiroidismo. La primera opción terapéutica del prolactinoma es la
farmacológica con dopaminérgicos, lo que prácticamente ha eliminado la cirugía. Los
dopaminérgicos suprimen la síntesis y secreción de prolactina con la consecuente
normalización del eje hipotálamo-gonadotrópico. Los dopaminérgicos son efectivos para
inducir la ovulación y favorecen la consecución de embarazo. Para cualquier dimensión del
prolactinoma se usan los dopaminérgicos durante uno a dos años y generalmente después de
suspenderlos se puede esperar que el tumor se reduzca de tamaño y se corrija la
hiperprolactinemia. En contraste, en el hombre generalmente se encuentran
macroprolactinomas, mayores de 10 milímetros con extensión extraselar acompañándose de
síntomas neurológicos; sin embargo, también responden favorablemente a los
dopaminérgicos.

Gac Méd Méx Vol. 140 No. 5, 2004


Referencias
1. Schlechte JA. Prolactinoma. N Engl J Med 2003;349:2035-2041.
2. Zárate A, Canales ES, Jacobs LS, Soria J, Daughaday WH. Restoration of ovarian
function in patients with the amenorrhea-galactorrhea syndrome after long-term therapy
with L-Dopa. Fertil Steril 1973;24:340.
3. Tyson JE, Carter JN, Andreassen B, Huth J, Smith B. Nursing mediated prolactin
and luteinizing hormone secretion during puerperal lactation. Fertil Steril 1978;30:154.
4. Schlechte JA, Sherman BM, Chapler FK, VanGilder J. Long-term followup of women
with surgically treated prolactin-secreting pituitary tumors. J Clin Endocrinol Metab
1986;62:1296-301.
5. Losa M, Mortini P, Barzaghi R, Gioia L, Giovanelli M. Surgical treatment of
prolactin-secreting pituitary adenomas: early results and long-term outcome. J Clin
Endocrinol Metab 2002;87:3180-3186.
6. Zárate A, Canales ES, Cano C, Pilonieta CJ. Follow-up of patients with prolactinomas
after discontinuation of long-term therapy with bromocriptine. Acta Endocrinol
1983;104:139-42.
7. Zárate A, Canales ES, Alger M. The effect of pregnancy and lactation on pituitary
prolactin secreting tumors. Acta Endocrinol 1979;92:407-11.
8. Bevan JS, Webster J, Hburke J, Scanlon MF. Dopamine agonists and pituitary tumor
shrinkage. Endocr Rev 1992;13:220-240.

11.- Algunas de las alteraciones que podemos encontrar en el síndrome metabólico según los
criterios del ATP III son:

a) Infertilidad, irregularidades menstruales.


b) Infarto agudo del miocardio o angina.
c) Arritmias o insuficiencia hepática.
d) Hipertensión e hipertrigliceridemia.
Existen varios criterios para definir el síndrome metabólico (criterios de la OMS, de la
Asociación Americana de Endocrinológos Clínicos, del ATP III). Los criterios del ATPIII
son los más prácticos. Se hace diagnóstico de sx metabólico cuando se encuentran 3 o más
de los siguientes criterios: 1) perímetro abdominal > 90cm en las mujeres o de 102 en los
hombres. 2) colesterol HDL en plasma < 40 mg/dl en hombres y < 50 mg/dl en mujeres. 3)
Triglicéridos plasmáticos ≥ 150 mg/dl, 4) presión arterial elevada ≥ 130/85 mm Hg. 5)
glucosa sanguínea elevada ≥ 100 mg/dl.

Grundy SM, Cleeman JI, Bairey Merz CN, Brewer HB, Clark LT, Hunninghake DB,
Pasternak RC, Smith SC, Stone NJ. Implications of recent clinical trials for the
Narional Cholesterol Educations Program Adult Treatmente Panel III Guidelines.
Circulation 2004; 110:227-239.

12.- Es la patología que asocia distensión de los espacios aéreos respiratorios distales a los
bronquíolos terminales acompañada de destrucción de los tabiques alveolares:

a) Bronquitis crónica.
b) Enfisema.
c) Asma.
d) Asbestosis.

Enfisema Pulmonar: Es definido como una condición del pulmón caracterizado por
agrandamiento permanente y anormal de los espacios aéreos dístales a los bronquiolos
terminales acompañado de destrucción de la membrana alveolo-capilar.

Manifestaciones Clínicas:

TOS: Representa el síntoma mas frecuente, principalmente en pacientes con BC, suele ser
tos en accesos de predominio nocturno.

Expectoración: Es abundante, de color blanquecino y solo en caso de infección de aspecto


purulento.

Manifestaciones Clínicas:

Disnea: Usualmente es el síntoma de presentación principalmente en los pacientes con


Enfisema Pulmonar. Es por lo general una disnea evolutiva de grandes a pequeños esfuerzos.
Radiografía: Las imágenes que sugieren Enfisema Pulmonar son:

a.- Aumento de volumen pulmonar.


b.- Corazón en “gota”.
c.- Abatimiento Diafragmático.
d.- Aumento de aire retroesternal.
e.- Bulas enfisematosas.

• 1.- Weinberger SE. Neumología. 2a. Edición, Interamericana McGraw-Hill,


Pennsylvania, EUA, 1992.
• 2.- Khan MC, Linch III JP. Diagnóstico y tratamiento de las enfermedades
pulmonares. Editorial AUROCK, México, 1998.
• 3.- Fishman AP, Elías JA, Fishman JA, Grippi MA, Kaisser LR, Señor RM. Pulmonary
Diseases and disorder. 3a. Edición McGraw-Hill, EUA, 1998.

13.- Algunos de los medicamentos que pueden causar hiperprolactinemia son:

a) Prazocin y captopril.
b) Metoclopramida y antidepresivos tricíclicos.
c) Ácido nadilíxico y fenazopiridina.
d) Hormonas tiroideas y metimazol.

MEDICAMENTOS QUE CAUSAN HIPERSECRECIÓN DE PROLACTINA


Estos medicamentos ingeridos aumentan la secreción de prolactina mediante una inhibición
dopaminérgica.
a) Tranquilizantes: Disminuyen el nivel de catecolaminas hipotalámicas o bien bloquean los
receptores dopaminérgicos pituitarios.
• Fenotiazinas: (Clorpromazina, trimeprazina, tioridazina, etc.),
• Diazepóxidos:
• Haloperidol
• Metoclopramida
• Antidepresivos tricíclicos
• La morfina, heroína
• Sulpirida
• Domperidona
• Meprobamatos
• Isoniazidas
• Anfetamina
b) Antidepresivos: La reserpina y la metildopa actúan disminuyendo el contenido
hipotalámico de la hormona dopamina además de llevar a cabo estimulación serotoninérgica.
c) Anticonceptivos hormonales: Efecto lactótrofo pituitario, la galactorrea puede aparecer
durante o una vez suspendido el medicamento.

MEDICAMENTOS QUE DISMINUYEN LA SECRECIÓN DE PROLACTINA:


a) Agonistas dopaminérgicos: apomorfina, bromoergocriptina, LSD, l-dopa, quinagolida,
serocriptina
b) Inhibidores del sistema de la Monoaminooxidasa: pargilina
c) Hormonas: Glucocorticoides
d) Otros: Nicotina.
Comparato M.R. Terapéutica Hormonal en Ginecología. 1988. Editorial "El Ateneo".
Capítulo 2 Hormonas sexuales. Páginas 36-37 Capítulo 13 Terapéutica hormonal en
endocrinología ginecológica Páginas 207-216.
Farreras P.Valenti y Rozman C. Medicina Interna.1982. Décima Edición. Ediciones
Doyma. Capítulo 15 Endocrinología. Páginas 1805, 1806, 1811, 1812, 1814.
Fischbach F.T. Manual de Pruebas Diagnósticas. 1997. McGraw-Hill Interamericana
5ª edición Capítulo 6 pruebas químicas Páginas 392-393.

Fuchs, F. y Koppler A. Endocrinología de la Gestación. 1982. Segunda edición. Salvat


Editores,S.A. Capítulo 12 Prolactina Humana. Páginas 249-272.
Henry J.B. Diagnostico y Tratamiento Clínicos. 1993. 9a edición. Masson-Salvat
Medicina Capítulo 15 evaluación de la Función Endocrina.. Págs.320-321.

14.- Paciente femenino de 28 años de edad, que acude a consulta por presentar dolor en el
recto, sin encontrarse una causa orgánica después de múltiples exploraciones médicas.
Además refiere que desde hace 4 años ha presentado vómitos, dolor abdominal
generalizado, palpitaciones, mareos, disfagia, visión borrosa, dolor en los miembros
inferiores, dismenorrea y dispareunia. Se observa triste y ansiosa. ¿Cuál es el diagnóstico
más probable?

a) Trastorno de ansiedad crónico.


b) Depresión crónica con somatización.
c) Trastorno de somatización.
d) Trastorno de conversión.

López- Ibor J J, Valdés M M. Manual diagnóstico y estadístico de los trastornos


mentales (DSM- IV). Masson 2005. 545 – 574. El Trastorno de somatización se
caracteriza por la presencia de muchos síntomas somáticos que no pueden explicarse por
los hallazgos físicos o de laboratorio. Comienza antes de los 30 años, puede perdurar
durante años, es crónico y va asociado a malestar psicológico, a un deterioro del
funcionamiento social y laboral y a la búsqueda excesiva de ayuda médica. Para hacer el
diagnóstico deben presentarse dolor en cuatro zonas del cuerpo, dos síntomas
gastrointestinales, un síntoma sexual y un síntoma pseudoneurológico. La ansiedad y la
depresión son las patologías psiquiátricas más prevalentes.
15.- En los estudios epidemiológicos existe un modelo retrospectivo, observacional,
comparativo, que parte del efecto a la causa, que es útil cuando se investigan
enfermedades de baja incidencia, que no expone a riesgo a ninguno de los sujetos
estudiados y que ofrece un índice conocido como riesgo relativo (Odds Ratio). Ese diseño es:

a) La serie de casos.
b) El reporte epidemiológico.
c) Ensayo clínico controlado.
d) Casos y controles.

Córdova VH, Jiménez J, Jiménez MC. Manual de diseño metodológico en investigación


clínica. ULSA – UAPY 2001. Pag 30 – 31. Los estudios de casos y controles son
retrospectivos, de observación y comparativos. A diferencia de los estudios de Cohorte,
parten en esencia desde el efecto y hacia atrás buscan identificar causas o factores de
riesgo o exposición. Al grupo de personas que tienen el fenómeno o enfermedad se le
denomina “casos” y se compara con otro grupo de individuos que no tienen el fenómeno o
enfermedad y se les denomina “controles” Este tipo de diseño es particularmente útil
cuando queremos estudiar a pacientes o poblaciones con alguna enfermedad que se
presentó mucho tiempo después de haber ocurrido la exposición o cuando queremos
investigar los factores de riesgo de enfermedades poco frecuentes o con causas múltiples.
En estos estudios se calcula el riesgo relativo (Odds Ratio) y se interpreta en razón al valor
de 1, esto es, menos de uno es poco probable que ocurra el riesgo relativo y más de uno es
probable que sí ocurra.

16.- La enfermedad sistémica mas comúnmente asociada a queratoconjuntivitis sicca es:

a) Lupus.
b) Granulomatosis de Wegener.
c) Artritis reumatoide.
d) Espondilitis anquilosante.

Queratoconjuntivitis Sicca
La queratoconjuntivitis sicca (QCS – Figura 1) es la manifestación más frequente de la AR
ocular caracterizada por una falta de lubricación lo que trae como consecuencia la
sensación de cuerpo extraño o arenilla en los ojos empeorada por actividades que
disminuyan la frecuencia de parpadeo (la lectura y el uso del computador). El test de
Schirmer generalmente da valores disminuidos. El manejo en las formas leves es con
lubricantes tópicos así como el control de otras enfermedades asociadas como meibomitis o
rosacea. En formas moderadas el uso de los tapones lagrimales ha revolucionado el
tratamiento permitiendo conservar las pocas lágrimas disponibles que contienen factores
de crecimiento fisiológicos y para los casos más severos, ocasionalmente es necesaria una
tarsorrafia parcial.
1. Sall K, et al. Two Multicenter, Randomized Studies of Efficacy and safety of
Cyclosporine
Ophthalmic Emulsion in Moderate to Severe Dry Disease. Ophthalmology
2000:107(4):631-639.
2. Donato BF, et al. Senile Atrophy of the Human Lacrimal Gland: The contribution chronic
inflammatory disease. Br Ophtalmol 1984;68:674-80.
3. Schein O, et al. Prevalence of dry eye among the elderly. Am J Ophtalmol 1997;
124:732-8.
4. Lemp MA, et al. Recent development in dry eye management. Ophthalmology 1987;
94:1299-1304.
5. Lubniewsky AJ, et al. Diagnosis and management of dry eye and oculary sulfatance
disorders.
Ophthalmogy Clinic of North America 1990;3:575-94.
6. Makie I, et al. Diagnostic implication of tear protein profiles. Br J Ophtalmol
1984;68:332.

17.- En la litiasis renoureteral: ¿Cuáles son los cálculos radiolúcidos?

a) De calcio.
b) De estruvita.
c) De ácido úrico.
d) De cistina.

Bruce E. Jarrell, R. Anthony Carabasi, Nacional Medical Series for Independent Study.
Wiliams & Wilkins, 3rd Edition: 451-475.
Los cálculos que sepueden presentar en la vía urinaria son formados por calcio, ácido úrico y
cistina, los únicos radiolucidos son los de ácido úrico que representan un reto diagnóstico.

18.- Una de las siguientes condiciones permite diferenciar la pielonefritis aguda de la


cistitis aguda:

a) Bacteriuria.
b) Fiebre de más de 38,5ºC.
c) Leucocituria.
d) Hematuria.
La sintomatología de una cistitis aguda se caracteriza por disuria, polaquiuria, piuria y
bacteriuria, y a menudo hematuria. El uso de espermicidas o de un diafragma como
anticonceptivo son conocidos factores de riesgo, y suele existir una relación temporal con
las relaciones sexuales o con un cambio en la situación hormonal, por ejemplo durante el
embarazo. La pielonefritis aguda se caracteriza por un dolor espontáneo o a la percusión uni
o bilateral en fosa renal, fiebre con o sin escalofríos, náuseas, vómitos y malestar general.
Suele haber disuria (oliguria, polaquiuria, urgencia miccional), aunque no necesariamente.

BIBLIOGRAFÍA

1. NABER, K. y cols. Guidelines on the Management of Urinary and Male Genital Tract
Infections, EAU, März 2006.)
2. PERTEL, P.E., HAVERSTOCK, D. BJU Int 2006; 98: 141 – 147.
3. TALAN, D.A. y cols. JAMA 2000; 283: 1583 – 1590.

19.- Datos de sospecha de complicaciones de otitis media aguda.

a) Parálisis facial, aumento de volumen retroauricular y déficit neurológico.


b) Fiebre,otorrea y otalgia.
c) Cefalea e hipoacusia conductiva.
d) Astenia y adinamia.
e)

Dentro de los datos de complicación de otitis media aguda se encuentran: mastoiditis,


petrositis, laberintitis, parálisis facial, meningitis, abscesos cerebrales, subdurales y
meníngeos y tromboflebitis de senos venosos.

Manual CTO de Medicina y Cirugía. Séptima Edición. McGraw Hill, pgs 1242.

20.- Una mujer de 25 años de edad presenta fiebre y artritis inflamatoria que afecta a las
articulaciones metacarpofalángicas e interfalángicas proximales ¿Cuál de las siguientes
manifestaciones es muy sugestiva de lupus eritematoso diseminado y no de artritis
reumatoide?

a) Sedimento urinario activo (eritrocitos, leucocitos, cilindros celulares, sin bacterias).


b) Artritis inflamatoria de articulaciones metacarpofalángicas e interfalángicas.
c) Derrame pleural de la radiografía de tórax.
d) Pruebas de funcionamiento hepático anormales.
Allen R. M. MMS Medicina Interna. 5ª. Edición. National Medical Series. Mc. Graw Hill.
2006. (capítulo 10IIG;VIIG l;cuadro 10-10). Un sedimento urinario activo sugiere
glomerulonefritis, dato común en pacientes con lupus eritematoso diseminado, pero no en
quienes tienen artritis reumatoide. Pueden encontrarse artritis de articulaciones
metacarpofalángicas e interfalángicas proximales derrames pleurales y anemia en
cualquiera de las dos enfermedades. Las anomalías de la función hepática son atípicas en el
lupus eritematoso sistémico, pero comúnmente son resultado de disfunción hepática
relacionada con fármacos (p. ej., por antiinflamatorios no esteroideos en cualquier
enfermedad.

21.- Un paciente de 65 años presenta dolor, calor y tumefacción en la rodilla derecha. ¿Cuál
de los siguientes hallazgos sería más útil para establecer el diagnóstico de pseudogota en
este enfermo?

a) Crecimiento de articulaciones interfalángicas proximales e interfalángicas distales.


b) Altas concentraciones séricas de ácido úrico.
b) Cristales con birrefringencia negativa del líquido de la rodilla.
d) Calcio en el menisco en una radiografía de la rodilla afectada.

Allen R. M. MMS Medicina Interna. 5ª. Edición. National Medical Series. Mc. Graw Hill.
2006. (capítulo 10 IV B 5 b). El hallazgo de un menisco calcificado en la radiografía de la
rodilla afectada es un dato diagnóstico de enfermedad por dihidrato de pirofosfato de
calcio, que sugiere que la inflamación de la rodilla es causada por seudogota. El crecimiento
de las articulaciones interfalángicas proximales y distales sugiere sólo osteoartritis y no
una causa específica. El aumento sérico de urato se relaciona con gota. El aumento de
cristales con birrefringencia en examen de luz polarizada compensada roja, de líquido
sinovial, es específico para el diagnóstico de seudogota. Hay muchas causas de derrame
inflamatorio aparte de esta entidad.

22.- En nuestro país consideramos en los enfermos con diabetes mellitus tipo 2 como un
criterio de buen control metabólico el siguiente:

a) Glucemia en ayuno < de 110 (mg/dl), glucemia postprandial de 2 hrs. < 140
(mg/dl), colesterol total < 200 (mg/dl), triglicéridos en ayuno < 150 (mg/dl) y
colesterol HDL > 40 (mg/dl).
b) Glucemia en ayuno < de 110 (mg/dl), glucemia postprandial de 2 hrs. < 200 (mg/dl),
colesterol total < 200 (mg/dl), triglicéridos en ayuno < 150 (mg/dl) y colesterol
HDL > 40 (mg/dl) .
c) Glucemia en ayuno > de 110 (mg/dl), glucemia postprandial de 2 hrs. > 200 (mg/dl),
colesterol total < 200 (mg/dl), triglicéridos en ayuno > 150 (mg/dl) y colesterol
HDL > 40 (mg/dl).
d) Glucemia en ayuno < de 90 (mg/dl), glucemia postprandial de 2 hrs. < 200 (mg/dl),
colesterol total < 200 (mg/dl), triglicéridos en ayuno < 150 (mg/dl) y colesterol
HDL > 40 (mg/dl).
MODIFICACION a la Norma Oficial Mexicana NOM-015-SSA2-1994, Para la prevención,
tratamiento y control de la diabetes mellitus en la atención primaria para quedar como
Norma Oficial Mexicana NOM-015-SSA2-1994, Para la prevención, tratamiento y control
de la diabetes.

APENDICE NORMATIVO E
METAS BASICAS DEL TRATAMIENTO Y CRITERIOS PARA EVALUAR EL GRADO
DE CONTROL DEL PACIENTE

Metas del tratamiento Bueno Regular Malo

Glucemia en ayunas (mg/dl) <110 110-140 >140

Glucemia postprandial de 2 h. <140 <200 >240


(mg/dl)

Colesterol total (mg/dl) <200.0 200-239 >240

Triglicéridos en ayuno (mg/dl) <150 150-200 >200

Colesterol HDL (mg/dl) >40 35-40 <35

P.A. (mm de Hg) <120/80 121-129/81-84 >130/85**

IMC <25 25-27 >27

HbA1c* <6.5%mg/dl 6.5-8%mg/dl >8%mg/dl

* En los casos en que sea posible efectuar esta prueba. ** Es necesario un control estricto
de la P.A. para reducir el riesgo de daño renal. Si el paciente fuma una meta adicional es
dejar de fumar.

23.- ¿Cuál es la causa más frecuente en el adulto de insuficiencia testicular adquirida?

a) Orquitis viral.
b) Orquiectomía.
c) Cáncer testicular.
d) Disgnesia gonadal.
La orquitis víral puede deberse al virus de la parotiditis epidémica, virus ECHO, virus
linfocítico de la coriomeningitis y arbovirus del grupo B. La orquitis ocurre hasta en una
cuarta parte de los varones adultos que sufren parotiditis. Es unilateral en casi dos tercios
y bilateral en los restantes. El análisis del semen solamente se normaliza en 33% de los
varones que sufren orquitis bilateral.
Bashin S, Jameson JL. Trastornos de los testículos y del sistema reproductor masculino. En
Jameson JL (ed): Harrison. Endocrinología. 1a ed. Madrid. MacGraw-Hill España, 2006: 184.

24.- Durante su consulta en la unidad de Medicina Familiar recibe a un paciente de 60 años


de edad que asegura ser hipertenso de más de cinco años de evolución, sin agudizaciones
además de ser portador de una fibrilación auricular, motivo por los cuales recibe
propranolol en dosis de 100 mgs cada mañana. Al revisar su electrocardiograma usted
espera encontrar:

a) Ausencia de onda P, complejo QRS normal e intervalos R-R diferentes.


b) Ausencia de onda P, complejo QRS ensanchado, diferencia constante de los intervalos R-
R.
c) Presencia de onda P, complejo QRS ensanchado y eje rotado a la derecha.
d) Ritmo nodal con disociación A-V.

Guadalajara J. Cardiología. Sexta Edición 151 – 152 La fibrilación auricular es la


arritmia cardiaca más frecuente. La despolarización caótica y desordenada de las aurículas
por múltiples ondas en simultánea, trae como consecuencia que se pierda la función
mecánica de la contracción auricular. Estos dipolos de activación múltiple y desordenados
alcanzan al nodo A-V y penetran en él, algunos pasan hacia el Haz de His mientras que otros
no lo alcanzan, debido a la penetración parcial del nodo por conducción decreciente
completa, esto se conoce como, conducción oculta; así, la rápida penetración de los
estímulos auriculares favorece la aparición de conducción oculta, la cual afecta en forma
impredecible el periodo refractario del nodo. El trazo electrocardiográfico típico es la
ausencia de onda P, complejo QRS normal e intervalos R-R diferentes.

25.- Ante un paciente con diagnóstico de trastorno obsesivo-compulsivo, usted decide


indicar el siguiente fármaco por ser el principal tratamiento de elección:

a) Antipsicóticos.
b) Antidepresivos inhibidores de la recaptación de noradrenalina.
c) Antidepresivos inhibidores de la recaptación de serotonina.
d) Benzodiacepinas.
El abordaje farmacológico del TOC se basa en los inhibidores selectivos de la recaptura de
serotonina (ISRS), medicamentos que han resultado ser efectivos y seguros. Como grupo,
los ISRS son igual de eficaces que la clorimipramina, pero producen menos efectos
secundarios y, por lo tanto, mejor tolerancia y mejor apego al tratamiento. La efectividad
antiobsesiva parece ser independiente de su actividad antidepresiva.
Entre los factores predictores de respuesta al tratameinto, se ha propuesto que los altos
puntajes en las compulsiones predicen una mala respuesta al tratamiento. Para los casos
refractarios y resistentes o cuando hay síntomas de comorbilidad, se han utilizado, con
relativo éxito, combinaciones con diferentes IRS, con benzodiacepinas, o bien, con
potenciadores como el litio, o antipsicóticos, como la risperidona y el haloperidol. El uso del
carbonato de litio es controvertido, aunque parece ser útil como potenciador a largo plazo
entre 15 y 30% de los pacientes. La combinación de ISRS con antipsicóticos comenzó a
utilizarse en los pacientes con síntomas psicóticos, aunque ahora se combinan también en
los pacientes resistentes.
Tratamiento farmacológico del TOC
Cristina Lóyzaga*
Humberto Nicolini*
*División de Investigaciones Clínicas. Instituto Nacional de Psiquiatría.
Ramón de la Fuente. Calzada México-Xochimilco, 101, San Lorenzo
Huipulco, 14370, México D.F.
Primera versión: 14 de septiembre de 2000.
Aceptado: 26 de septiembre de 2000.
AKERMAN DL, GREENLAND S, BYSTRISKY A: A side effects as predictors of drug
response in obsessive compulsive disorder. J Clin Psychopharmacol, 19(5):459- 465, 1999.
2. ALARCON R, WESLEY J, SPITTER D: A predictive study of obsessive-compulsive
disorder response to clorimipramine. J Clin Psycopharmacol, 13(3):210-213, 1993.
3. ALDERMAN J, WOLKOW R, CHUNG M, JOHNSTONE
HF: Sertraline treatment of child and adolescent with OCD or depression. J Am Acad
Child Adolesc Psychiatry,
37(4):386-394, 1998.
4. BARR L, GOODMAN W, PRICE L: The serotonine hypotheses of obsessive-compulsive
disorder: implications of pharmacological challenge studies. J Clin Psy, 53 (Supl)
4:17-28, 1992.

26.- Un paciente con carcinoma pulmonar presenta náuseas, vómitos y letargo y se


encuentra que tiene una concentración sérica de calcio de 13.4 mg/100 ml. ¿Cuál de los
siguientes agentes debe ser el primer paso para el tratamiento?

a) Etidronato intravenoso.
b) Mitramicina intravenosa.
c) Glucocorticoides intravenosos.
d) Solución salina y furosemida intravenosos.
Allen R. M. MMS Medicina Interna. 5ª. Edición. National Medical Series. Mc. Graw Hill.
2006. (capítulo 9 IIIA 6 a, b, d, 7 c (1), (3)). La hipercalciemia causada por otras
enfermedades aparte del hiperparatiroidismo puede tratarse con solución salina y
furosemida intravenosos. El reemplazo de líquidos con solución salina intravenosa, seguido
por diuresis forzada con solución salina y furosemida intravenosas, es una forma rápida y
segura de disminuir el calcio sérico y debe intentarse primero. Pueden añadirse
pamidronato, mitramicina o calcitonina si se necesita disminuir aún más la concentración de
calcio. Los glucocorticoides son eficaces para tratar la hipercalciemia causada por exceso
de vitamina D, sarcoidosis y algunas neoplasias malignas hemáticas, pero no disminuyen el
calcio sérico en la mayor parte de los casos de hipercalciemia relacionada con tumores
sólidos.

27.- Una mujer de 20 años de edad presenta trombosis venosa profunda de nueva
aparición en la extremidad inferior izquierda. Tiene antecedentes de trombocitopenia leve
y dos abortos; fue tratada para sífilis hace dos años debido a una prueba de reagina rápida
del plasma positiva, aunque el anticuerpo antitreponémico fue negativo. ¿Cuál de los
siguientes trastornos es más probable que represente el conjunto de su historia clínica ?

a) Síndrome de anticuerpos antifosfolípido.


b) Lupus eritematoso diseminado (SLE) .
c) Síndrome de anticuerpo Ro.
D) Arteritis de Takayasu.

Allen R. M. MMS Medicina Interna. 5ª. Edición. National Medical Series. Mc. Graw Hill.
2006. (capítulo 10VIIFl,2c, G3b). Esta paciente tiene varias manifestaciones sutiles de
síndrome de anticuerpo antifosfolípido, que en conjunto hacen de éste un diagnóstico
probable. La prueba de reagina plasmática rápida (RPR) positiva, con resultados negativos
de la prueba de treponemas, probablemente refleja anticuerpos que producen reacción
cruzada a la cardiolipina o a componentes fosfolípidos de antígenos treponémicos. La
trombocitopenia es frecuente en esta situación, debido a las interacciones de plaquetas y
células endoteliales y a la coagulación inducida por los anticuerpos. Los abortos pueden
deberse a coagulación en vasos placentarios pequeños. Las trombosis venosas profundas o
incluso la coagulación en arterias mayores pueden ser causadas por hipercoagulabilidad
consecuente. No hay otras pruebas de lupus eritematoso diseminado, aunque pueden
encontrarse anticuerpos antifosfolípido en alrededor de 33% de los pacientes. Tampoco se
describen datos de síndrome de anticuerpo Ro o enfermedad indiferenciada del tejido
conjuntivo. La arteritis de Takayasu se presenta en mujeres asiáticas jóvenes, pero no hay
datos de isquemia de grandes vasos arteriales característica de esta enfermedad.
28.- En relación con los signos de muerte la ausencia de los latidos cardiacos sobre los
focos de auscultación se le conoce como signo de:

a) Winslou.
b) Neumatoscopía.
c) Bouchut.
d) Icard.

DIAGNÓSTICO DE DEFUNCIÓN

A través de la historia se ha buscado el método más idóneo para diagnosticar la defunción,


así encontramos que lo clásico es buscar signos vitales en el paciente en cuestión, o por
medio de métodos más elaborados. Actualmente en México nos rige la ley General de Salud,
y en sus artículos 317 y 318 especifica como se determina la certificación de la muerte.
Como antecedentes se mencionan algunos métodos que se utilizaron para certificar la
muerte.

CIRCULATORIOS

Prueba de Icard. La cual consiste en inyectar cinco mililitros de una solución de


fluoresceina por vía intravenosa. Lo que da como resultado en el paciente vivo, una
coloración amarilla en la piel y un tono verdoso esmeralda en el segmento anterior de los
ojos en un tiempo no mayor de cinco minutos. Lo cual quiere decir que aun existe circulación
en el organismo. Esta pruebe es determinante.

Prueba de Bouchut. Consiste en auscultar los latidos cardiacos durante doce a veinte
minutos y puede dar resultados falsos por factores tales como uso inadecuado del
estetoscopio, capacidad auditiva deficiente del medico explorador, etc.
Esta prueba no es determinante.

Signo de Magnus. Consiste en ligar un dedo desde su base, en caso de existir circulación
este se tornara cianótico.

Signo de Middeldorf. Se introduce una aguja tipo tuy a nivel del apex del corazón, si
presentara movimiento aún. existen latidos cardiacos.

Vargas Alvarado E. Medicina Legal. ED. Trillas, 2000, México. (Cáp. 8 p. 91)
29.- Las artropatías que predisponen con mayor frecuencia a artritis séptica son:

a) La espondilitis anquilosante y la gota.


b) La osteoartrosis y el lupus eritematosos sistémico.
c) La artritis reumatoide y la espondilitis anquilosante.
d) La artritis reumatoide y la osteoatritis.

Artritis séptica

La artritis infecciosa es una reacción inflamatoria de la cavidad articular debido a


una infección por diferentes microorganismos, siendo los estafilococos, estreptococos,
neumococos, gonococos y pseudomonas, los mas frecuentes. La sepsis ocasionada por
estos gérmenes es conocida como infección piógena.
Existen una serie de factores que predisponen a la artritis infecciosa:

1. Enfermedad articular preexistente (artritis reumatoide, osteoartritis)


2. Enfermedad crónica sistémica (diabetes, LES, neoplasias)
3. Infecciones intraarticulares
4. Drogadicción intravenosa
5. Administración de corticosteroides
6. Punción articular
7. Traumas.

Reumatología Manuel Lombas García.


Revista Cubana de Reumatología | Revistas Medicas Cubanas .
Dr. Mtz Larrarte JP .
Dra. Reyes Pineda Y.
Servicio Nacional de Reumatología.

30.- Masculino de 56 con derrame pleural. Se realiza una toracocentesis y se analiza el


fluido revelando que tiene una diferencia de proteínas con el suero de 0.9 y diferencia de
DHL con suero de 1. Este fluido es un:

a) Trasudado.
b) Exudado.
c) Derrame.
d) Infiltrado.
Toracentesis y análisis del líquido pleural. La toracentesis diagnóstica requiere
menos de 30 ml de líquido. En los derrames enquistados es útil la ecografía para localizar
con precisión el líquido, y con ello hacer la toracentesis más fácil y sin riesgos (Ver guía
para drenaje y succión pleural).

Los derrames pleurales se clasifican en trasudados y exudados. Un trasudado es un filtrado


de plasma que resulta del aumento de la presión hidrostálica o de la alteración de la
permeabilidad capilar. Los trasudados se asocian con insuficiencia cardiaca congestiva,
sindrome nefrótico, cirrosis y condiciones de sobrecarga de volumen.

El exudado es un líquido rico en proteínas resultante de una inflamación local o por una falla
en la eliminación de proteínas por los linfáticos o ambos mecanismos. Los exudados se
producen en infecciones colagenopatías y neoplasias.

Existen criterios para la diferenciación entre trasudados y exudados (los exudados deben
cumplir al menos uno de los siguientes criterios):

a. Relación de proteína pleural/sérica > 0.5.


b. Relación de LDH pleural/sérica > 0.6.
c. LDH pleural > 200 Ul.
d. Estos criterios tiene significancia diagnóstica con una sensibilidad del 98% y
especificidad de 77%.

Otros criterios útiles para el diagnóstico son:

a. Colesterol total en líquido pleural mayor de 60 mg/dl


b. Relación de colesterol pleural/sérico mayor 0.4

Otro análisis del líquido pleural de importancia es la concentración de glucosa. Un resultado


bajo (< 60 mg/dl) en el líquido pleural es sugestivo de empiema, neoplasia, TBC, LES o
pleuresía reumática.

El pH normal es de 7.60 encontrándose menor a 7.30 en las mismas entidades patológicas


descritas para la glucosa, y en la ruptura esofágica; en los derrames por neoplasia un pH
bajo se relaciona con menor sobrevida y menor respuesta a la pleurodesis química.

La medición de triglicéridos es útil ante la sospecha de quilotórax (> 110 mg/dl).

La medición de adenosina deaminasa permite la diferenciación de TBC pleural y neoplasia


cuando es mayor de 45 UI.
El recuento y la diferenciación celular ayudan al diagnóstico etiológico del derrame. Los
trasudados en general tienen menos de 1.000 leucocitos/ml; recuentos mayores a
10.000/ml se ven en derrames paraneumónicos, mayores de 50.000/ml en empiema. Los
derrames crónicos (TBC, neoplasia) tienen menos de 5.000/ml. La linfocitosis es indicativa
de TBC, neoplasia, linfoma, sarcoidosis, pleuresía reumática. Se encuentra predominio
neutrofílico en neumonía, embolía y pancreatitis.

Ante la sospecha de neoplasia, se debe solicitar una citología del líquido pleural, la cual
tiene una sensibilidad importante.

LECTURAS RECOMENDADAS

American College of Physicians. Diagnostic thoracentesis and pleural biopsy in pleural


effusions Ann Inter Med 103:799, 1985

Camacho Durán F, Restrepo Molina J. Enfermedades de la pleura. En: Fundamentos de


Medicina. Neumología. Tercera Edición.

Corporación para Investigaciones Biológicas. Medellín, 1986

Des Jardins T. Enfermedades pleurales En: Enfermedades Respiratorias. Terry Des


Jardins Editor.

Editorial El Manual Moderno SA México DF, 1993

Light RW, Mac Gregor M. The diagnostic separation of trasudates and exudates.

Ann Intern Med 77: 507, 1972.

Pacheco PM. Estudio del derrame pleural En: Enfermedades del Tórax. Fidel Camacho,
Jaime Páez, Carlos Awad Editores.

Ediciones Médicas Zambón. Santafé de Bogotá, 1992

Patiño JF, Arroyo de S. Guía práctica de toracentesis y de toracostomía cerrada (inserción


de tubo de tórax).

Trib Médica 89:161, 1994

Patiño JF, Arroyo de S. Guía para drenaje y succión pleural. Manejo del drenaje pleural.
31.-Ante un paciente que presenta soplo holosistólico que borra el 1er ruido y se irradia a
la axila, usted sospecha de:

a) Estenosis mitral.
b) Insuficiencia mitral.
c) Estenosis aórtica.
d) Insuficiencia aórtica.

INSUFICIENCIA MITRAL:

Aguda:

- Daño agudo al la valva o al aparato subvalvular (rotura del músculo papilar).

- Trauma directo, o intenso.

- Endocarditis.

Crónica.

- Fiebre reumática.

- Enfermedades degenerativas ( de la colágena)

Artritis reumatoide.

Lupus eritematoso sistémico (LES).

Síndrome de Marfan (hipersensibilidad en articulaciones, afecta mas a


hombre, son altos, flacos, luxación del cristalino, insuficiencia mitral,
aneurisma de la aorta, insuficiencia aortica)

Alteración fisiopatológica:

- Aumento de presión en AI.

- Dilatación de AI.

- Aumento de presión en venas y capilares y pulmonares.

- Sobrecarga diastólica del VI y este crece.

- Falla el VI.
Manifestaciones clínicas:

- Disnea.

Auscultación:

- El soplo borra el primer ruido.

- Soplo holosistólico (toda sístole).

- Se oye el tercer ruido.

Electrocardiograma ( ECG )

- Aumento de crecimiento de AI y del VI.

Rayos X.

- Cardiomegalia.

Crecimiento de AI del VI, una hipertensión venular pulmonar.

Eco cardiografía:

- Nos da el diagnostico definitivo

- Dimensiones de AI y del VI.

- Funcionamiento del VI.

Complicaciones:

- EAP.

- Trombosis.

- Endocarditis.

- Fibrilación auricular.

- Hipertensión pulmonar.
VALVULA CARACTERISTICAS CAUSAS AUSCULTACI COMPLICACIONES
ON
EST. MITRAL Área normal 4-6cm2 Es la mas Anomatopeya Dilatacion AI
hemoptizante de Durozies
Mas 2 cm asintomática Escape sinusal.

Mas 1cm moderada Arritmias.

Menos 1 cm severa
INSUF. Se oye tercer ruido Aguda::trauma Soplo EAP
MITRAL endocarditis holosistólico
Trombosis.
Crónica: F.
reumática, Fibrilación auricular.
enfermedad de
la colágena. Hipertensión
pulmonar.

DOBLE Fiebre Soplo Aumento presion de


LESION reumática holosistolico AI.
MITRAL
Disnea.

Disminución del
gasto cardiaco
ESTENOSIS Área de 3 cm2 Aorta bivalva. Soplo Dilatación
AORTICA protomesosis- posestenortica.
Reumatica. tólico.
Disnea que estas 2:
Aterosclerotica.
- Disminución
de contracción.

- Daño.

- Remodelación.

- Dilatación

- Izquierda.
ISUF. puede que el corazón f. reumática. Soplo
AORTICA sea suficiente y solo diastólico.
se adapte enferemdades
degenerativas, Volumen de
escape o
artritis aspirativo.
reumatoide,espo
ndilitis, sifilis. Ocurre el
fenómeno de
Yet.

BIBLIOGRAFIA
1. Otto CM, Evaluation and management of chronic mitral regurgitation. N Engl J
Med.2001;345:740-46.
2. Corin WJ, Monrad ES, Murakami T, et al. The relationship of afterload to
ejection performance in chroonic mitral regurgitation. Circulation 1987;76:59-67.
3. Tsutsui H, Sinale FG, Nagatsu M, et al. Effects of crónic b-adrenergic blockade
on teh left ventricular and cardiocyte abnormalities of chronci canine mitral
regurgitation. J Clin Invest 1994;93:2639-48.
4. Zile MR, Tomita M, Ishihara K, et al. Changes in diastolic function during
development and correction of chronic LV volume overload produced by mitral
regurgitation. Circulation 1993;87:1378-88.

32.- Acude con usted masculino de 52 años con datos de alcoholismo crónico, en éste
momento cursa con un ataque agudo de gota, usted indica el siguiente medicamento ya que
se considera el de primera elección para el tratamiento de esta enfermedad:

a) Colchicina.
b) Un antiinflamatorio no esteroideo.
c) Prednisona.
d) Alopurinol.

Tratamiento
Si bien las modificaciones favorables en el estilo de vida pueden ser útiles para descender
los niveles de ácido úrico en sangre, la dieta suele ser de valor moderado y habitualmente
los pacientes requieren tratamiento farmacológico.
Tratamiento del ataque agudo de gota
Antiinflamatorios no esteroides. En ausencia de contraindicaciones, los antiinflamatorios no
esteroides (AINE) representan la primera línea de terapia para aliviar los síntomas del
ataque agudo de gota. Históricamente se utilizó indometacina pero otros AINE también son
de utilidad (en cambio, debe evitarse el salicilato en dosis bajas). Lamentablemente, los
efectos adversos frecuentes de estos agentes limitan su uso. Los AINE deben utilizarse
con cautela en sujetos de edad avanzada, en pacientes con insuficiencia renal o cardíaca
congestiva o en sujetos con patología hepática o ulcerosa; lo mismo sucede en individuos
que reciben anticoagulación. Los inhibidores selectivos de la ciclooxigenasa 2, como el
etoricoxib, son igual de eficaces que los AINE pero se asocian con menos efectos adversos;
no obstante, en los últimos tiempos surgió preocupación por los posibles efectos adversos
cardiovasculares.

Agudelo CA, Wise CM. Crystal-associated arthritis. Clin Geriat Med 1999; 14: 495
Fam AG. Current therapy of acute microcristalline arthritis and the role of
corticosteroids. J Clin Rheumatol 1997; 3: 35.
Groff GD, Frank WA, Raddatz DA. Systemic steroid therapy for acute gout: a clinical trial
and review of the literature. Semin Arthritis Rheum 1998; 34: 1
Hill GL, Agudelo CA, Semble EL. Parenteral ACTH in the treatment of acute gout.
Arthritis Rheum 1991; 34: S145.
Axelrod D, Preston S. Comparison of parenteral adrenocorticotropic hormone with oral
indomethacin In the treatment of acute gout. Arthritis Rheum 1998; 31: 803.
Agudelo CA. Crystal deposition diseases. In Weisman MH, Weinblatt ME. Treatment
of the Rheumatic Diseases. Chapter 17. Pp 271- 1st Ed. Philadelphia, WB Saunders,
1995.
Ben-Chetrit, Levy M. Colchicine: 1998 update. Semin Arthritis Rheum 1998; 28: 48.
Kuncl RW, Duncan G, Watson D. et al. Colchicine myopathy and neuropathy. N Engl J
Med 1997; 316: 1562.

33.- Nombre que recibe la zona circundante con reborde negruzco, provocado por la herida
de un proyectil de arma de fuego.

a) El Anillo de enjugamiento.
b) La Cintilla erosiva.
d) El Halo de dispersión.
d) El Anillo de contusión.

En condiciones óptimas, es decir un OE provocado por un proyectil que ha incidido


perpendicularmente al plano dérmico, con ojiva aguzada y sobre zona de tejido blando, el
OE será circular, de diámetro menor al del proyectil y estará rodeado de una zona circular
de características contuso-equimótico-escoriativas cuya mayor intensidad estará ubicada
junto al borde del orificio atenuándose paulatinamente a medida que se aleja de él. Esta
zona se conoce con el nombre de “Zona de Enjugamiento” o “ANILLO DE FISCH” y estará
presente siempre en los OE de proyectiles de armas de fuego, siendo uno de los signos que
lo manifiestan. La forma (circular u ovoidal) y la centricidad del Anillo de Fisch con
respecto al OE (concéntrico o excéntrico),
Vargas Alvarado E. Medicina Legal. ED. Trillas, 2000, México. (Cáp.17 pp. 201-
202).

34.- En la esquizofrenia, los trastornos sensoperceptivos más frecuentes son las


alucinaciones de tipo:

a) Tactiles.
b) Visuales.
c) Auditivas.
d) Olfatorias.

Manual CTO 5ª edición, Psiquiatría, pág 25 Principios de Medicina Interna Harrison


15ª Ed. Pag 2988.
Las alucinaciones son trastornos de la percepción. Recuerda que aparecen sin objeto (o
estímulo). En la esquizofrenia los más frecuentes son los auditivos, “mandatorios” o que
“comentan cosas” sobre el paciente. También pueden aparecer los táctiles (tienen bichos) o
sensaciones en la piel. Otros pueden ser somáticos que les hace creer que alguna parte del
cuerpo cambia. Los olfatorios son raros, pero también pueden aparecer, siendo más típicos
de las crisis parciales complejas. Es importante que asocies alucinaciones visuales a
enfermedades “orgánicas”, por lo que en esos casos debes descartar patologías
endocrinológicas, neurológicas, tóxicas...Recuerda que en la esquizofrenia aparecen
trastornos de la forma (ecolalia, perseveración...) y contenido del pensamiento (ideas
delirantes).

35.- Masculino de 42 años, alcohólico con cirrosis y ascitis. Es hospitalizado por agitación y
comportamiento extraño. ¿Cuál de los siguientes hallazgos es el más útil para hacer el
diagnóstico de encefalopatía hepática?

a) Ictericia.
b) Asterixis.
c) Hemangiomas superficiales.
d) Signo de la ola positivo.
Braunwald E, fauci A, Kasper D, Hauser S, Longo D, jameson L. Harrison Principios de
Medicina Interna. 15° edición. Mc Graw Hill 2001. 2062.

La encefalopatía hepática es un síndrome neuropsiquiátrico caracterizado por alteraciones


de la conciencia y de la conducta, cambios de la personalidad, signos neurológicos
fluctuantes, asterixis o temblor aleteante y alteraciones electrocardiográficas
características. Se debe plantear el diagnóstico cuando se presentan cuatro factores: 1.
enfermedad hepatocelular aguda o crónica; 2. alteraciones de la atención y del estado
mental; 3. combinaciones variables de signos neurológicos, como asterixis, rigidez,
hiperreflexia,, Babinski y convulsiones; 4. un trazado electroencefalográfico de ondas
lentas, trifásicas, simétricas y de alto voltaje.

36.- La mayoría de los errores innatos del metabolismo se transmiten de manera:

a) Autonómica dominante.
b) Autonómica recesiva.
c) Recesiva ligada al X.
d) Dominante ligada al X.

HERENCIA AUTOSÓMICA RECESIVA

Las enfermedades genéticas autosómicas recesivas requieren que el gen mutado esté en
doble dosis, o sea en estado homocigoto; en consecuencia, el individuo afectado recibe un
gene mutado de cada padre heterocigoto que es fenotípicamente normal. Cada padre
heterocigoto para un gen mutado, tiene 50% de riesgo de transmitir ese gene. Por ello,
cuando dos heterocigotos se unen, 25% de sus hijos estarían teóricamente afectados. La
relación de hijos afectados y normales sería de 1:3.

Contrariamente a lo que ocurre en enfermedades autosómicas dominantes, los


padecimientos autosómicas recesivos tienen un patrón hereditario horizontal y pueden
encontrarse varios hermanos afectados. La consanguinidad aumenta considerablemente la
aparición de enfermedades recesivas y cuanto más próxima sea la relación familiar, mayor
será el riesgo de que ambos miembros de la pareja hayan heredado el gen anormal del
antepasado común. .La enfermedad autosómica recesiva más frecuente (1:2500 recién
nacidos vivos en poblaciones caucásicas) es la fibrosis quística, caracterizada por
trastornos pancreáticos, respiratorios y de la sudoración, cuyo gene (CFTR) ha sido
localizado en los brazos largos del cromosoma 7. Este gene codifica para una glicoproteína
de membrana que corresponde a un canal de cloro en las células epiteliales de los órganos
exócrinos. La presencia de una proteína mutada explica los cambios electrofisiológicos que
se producen al modificarse la permeabilidad de la membrana aumentando la viscosidad de
las secreciones de las glándulas exócrinas del organismo, alterando su función normal.

La mayoría de los errores innatos del metabolismo tienen un patrón de herencia autosómica
recesiva, como fenilcetonuria, galactosemias, mucopolisacaridosis (excepto la tipo II),
glucogenosis, albinismo, anemia de células falciformes, enfermedad de Tay-Sachs,
hiperplasia suprarrenal congénita, talasemias, etc

37.- ¿Qué porcentaje del peso corporal corresponde al espacio intracelular?

a) 20 al 10%
b) 70 al 80%
c) 30 a 40%
d) 20 a 25%

El compartimento intracelular forma del 30 al 40% del peso corporal total (65% del agua
corporal total). La mayoría del agua intracelular se encuentra alojada en la célula músculo-
esquelética, un porcentaje muy pequeño en el adiposito.

1. Bruce E. Jarrell, R. Anthony Carabasi, Nacional Medical Series for Independent


Study. Wiliams & Wilkins, 3rd Edition: 1-25.

38.- Principal causa de rinitis alérgica perenne:

a) Ácaros en polvo casero.


b) Pólenes.
c) Cucarachas.
d) Caspa

La Rinitis Alérgica se divide en estacional y perenne, ésta división es causada por la


presencia del alérgeno en el aire, así, si sólo se da la presencia de éste en una o dos
estaciones del año, por ejemplo los pólenes, se denomina RA estacional, pero si el alérgeno
se encuentra en el aire todo el año, por ejemplo en el polvo casero con predominancia de
ácaros, se llamará perenne. También puede ocurrir que un individuo tenga alergia a más de
un alérgeno, los cuales tengan presencia estacional pero en diferentes estaciones, que
pueden ser continuas o no, entonces a ésta presentación se denomina RA estacional dual
(33,34). En la bibliografía mundial se reporta que el porcentaje de pacientes que sufren RA
perenne es igual al 40 a 50% de personas que tienen RA (4,7).

International Rhinitis Management Working Group. International Consensus Report on


the Diagnosis and Management of Rhinitis. Eur J Allergy Clin Inmunol 1994; 49 (suppl).
234-36

39.- En un niño sano de 3 meses, el valor de la hemoglobina es:

a) 3-17 g/dL
b) 9.5-14 g/dL
c) > 15 g/dL
d) < 9 g/dL
Nelson, Tratado de Pediatría 15ª Ed., págs. 325-326.

Las cifras de Hemoglobina tienen una evolución muy típica durante la infancia. El recién
nacido de término (RNT) tiene una cifras de hemoglobina de 14-20 g/dL, el RN de muy bajo
peso tiene 1-2 g/dL menos. A partir de las 48h, comienza a descender la Hb. En el RNT
hacia las 12 semanas (3 meses) de vida se produce un descenso fisiológico de Hb,
alcanzando cifras mínimas de 9-11g/dL y hacia las 6 semanas en el recién nacido pretérmino
con cifras de 7-10 g/dL. A partir de esa edad, las cifras de Hb empiezan a ascender
paulatinamente, estabilizándose en la edad adulta. Ante niveles menores a lo esperado de
Hb, hablamos de anemia. La anemia en los primeros días generalmente es causada por
hemólisis de los glóbulos rojos y déficit de eritropoyetina, mientras que en el resto de la
infancia suele ser ferropénica.

40.- Usted realiza el diagnóstico de diabetes insípida, dicha patología se debe a


deficiencia de:

a) Mineralocorticoides y cortiso.
b) Glucocorticoides.
c) ACTH.
d) Hormona antidiurética.

La diabetes insípida (del lat. insipidus: «sin sabor») es por deficiencia absoluta o relativa
de vasopresina, o por resistencia a su efecto. Los pacientes presentan poliuria hipotónica a
pesar de que tienen elevados el sodio y la osmolaridad. Hay dos clases: diabetes insípida
verdadera o central y la diabetes insípida familiar ligada al sexo (nefrogénica).
La diabetes insípida es un trastorno en el que los valores insuficientes de hormona
antidiurética causan una sed excesiva (polidipsia) y una producción exagerada de orina muy
diluida (poliuria).
La diabetes insípida es el resultado de un déficit de la hormona antidiurética (vasopresina),
que es la encargada de limitar la producción excesiva de orina. Lo singular de esta hormona
es que el hipotálamo la produce y luego es almacenada hasta ser liberada en el flujo
sanguíneo por la hipófisis posterior. El trastorno puede también aparecer cuando una
concentración de hormona antidiurética normal está combinada con una respuesta anormal
de los riñones a la hormona, una afección denominada diabetes insípida nefrógena.

Bringhurst FR, Demay MB, Kronenberg HM. Disorders of Mineral Metabolism. In:
Kronenberg HM, Schlomo M, Polansky KS, Larsen PR, eds. Williams Textbook of
Endocrinology. 11th ed. St. Louis, Mo: WB Saunders; 2008: chap. 27.
Robinson AG, Verbalis JG. Posterior Pituitary. In: Kronenberg HM, Schlomo M,
Polansky KS, Larsen PR, eds. Williams Textbook of Endocrinology. 11th ed. St. Louis,
Mo: WB Saunders; 2008: chap. 9.
41.- El siguiente es un medicamento empleado en el tratamiento de la fibromialgia:

a) Metotrexate.
b) Alprazolam.
c) D-Peniciliamina.
d) Hidroxicloroquina.

Las benzodiazepinas como el alprazolam (0.5 mg a 1 gr) en conjunto con ibuprófeno ha


mostrado mejoría contra un grupo control/placebo presumiblemente normalizando los
niveles de receptores para imipramina que tienen que ver con la recaptación de serotonina.
Su principal problema es la posibilidad de generar adición.

Korszun A. Sleep and circadian rhythm disorders in fibromyalgia. Curr Rheumatol Rep
2000;2(2):124-30.
Roizenblatt S, Moldofsky H, Benedito-Silva AA, Tufik S. Alpha sleep characteristics in
fibromyalgia. Arthritis Rheum 2001;
44(1):222-30. Hedenberg-Magnusson B, Ernberg M, Kopp S. Presence of orofacial pain and
temporomandibular disorder in fibromyalgia. A study by questionnaire. Swed Dent J
1999;23(5-6):185-92.
McBeth J, Macfarlane GJ, Benjamin S, Silman AJ. Features of
somatization predict the onset of chronic widespread pain: results of a large population-
based study. Arthritis Rheum 2001; 44(4):940-6.
Pongratz D, Spath M. Fibromyalgia. Fortschr Neurol Psychiatr 2001;69(4):189-93.
Jeschonneck M, Grohmann G, Hein G, Sprott H. Abnormal microcirculation and
temperature in skin above tender points in patients
with fibromyalgia. Rheumatology (Oxford) 2000;39(8):917-21.

42.- Ante un probable infarto agudo al miocardio, ¿Cuál de los siguientes marcadores
tiene mayor sensibilidad para realizar éste diagnóstico?

a) CPK .
b) Transaminasa .
c) Troponina .
d) Deshidrogenada láctica.

Es muy conocida la falta de especificidad de la CPK. La elevación de la actividad de esta


enzima se produce tanto en el infarto de miocardio (IAM) como en afecciones
caracterizadas por un grado variable de necrosis muscular. Por ello en los últimos años se
han realizado, y se siguen realizando, considerables esfuerzos para encontrar nuevos
marcadores analíticos que sean capaces de diferenciar estos cuadros. Entre ellos, la
determinación de la isoenzima miocárdica de la CPK de forma cuantificada (CPK-MB-Masa)
resulta también poco específica ya que se eleva también en caso de necrosis de músculo
estriado; está descrita su elevación en traumatismos, rabdomiolisis, convulsiones1,
miopatías agudas y crónicas2, insuficiencia renal en diálisis3 e incluso en el ejercicio
intenso4,5; y lo mismo podemos afirmar de la mioglobina6.

Más recientemente se han determinado las troponinas T e I en sus isoformas específicas


de músculo cardiaco, que tienen una secuencia diferente de aminoácidos a las de músculo
estriado no cardiaco, lo que permite el desarrollo de inmunoensayos específicos. El
complejo de las troponinas T, C, e I está estrechamente unido al filamento de tropomiosina.
La T se encarga de la unión a tropomiosina; la C es iniciadora de la contracción tras unirse
al calcio, y la I se llama así por ser inhibidora de la contracción en reposo7. Esta
especificidad ha sido demostrada en varios trabajos que muestran valores normales de
troponina-I en procesos que cursan con necrosis de músculo estriado2,8.

Además de ser muy específicas, las troponinas son altamente sensibles en el infarto de
miocardio. Mair y col encuentran una sensibilidad del 100% para el diagnóstico si se hace la
determinación de troponina-I pasadas 6 horas del comienzo del dolor9. La cinética de estos
marcadores en pacientes con IAM puede resumirse de esta forma:

1. La CPK total comienza a elevarse a las 4-8 horas del comienzo de los síntomas, y
permanece elevada hasta que se normaliza a las 48-72 horas10.

2. La isoenzima MB de la CPK (CPK-MB) aparece en el suero tres horas después del


comienzo del IAM, con un pico máximo a las 18-20 horas, alcanzando valores 16 veces
superiores al normal. A partir de este punto desciende lentamente y persiste elevada al
menos 2 días11.

3. La troponina-I se eleva a partir de las 2-3 horas del comienzo de los síntomas, con un
valor máximo a las 16 horas. Desciende bruscamente hasta las 48 horas, y a partir de
entonces se produce un lento descenso; puede detectarse todavía el 7º-8º día11.

4. La mioglobina es la primera que se eleva. Da las cifras más altas de sensibilidad en el


plazo de dos horas del comienzo del dolor10 con respecto a la troponina-I y la CPK-MB.
Alcanza su pico a las 8 horas y desciende bruscamente a valores normales a las 18 horas11

43.- Femenino de 78 años de edad con antecedente de insuficiencia cardiaca congestiva


presenta angina de pecho. Sus medicamentos se ajustan con: furosemida, digoxina,
nitroglicerina y potasio complementario. Poco después presenta cefaleas pulsátiles
intermitentes. ¿Cuál de las siguientes medidas debe tomar primero el médico?

a) Realizar biopsia de arteria temporal.


b) Comenzar la administración de propranolol.
c) Comenzar la administración de ergotamina sublingual.
d) Suspender la nitroglicerina.
Allen R. M. MMS Medicina Interna. 5ª. Edición. National Medical Series. Mc. Graw Hill.
2006. (capítulo 11IV B 1 a (2), d (1) (a), 4 a-b). La nitroglicerina puede causar cefaleas
"vasculares" terebrantes; por tanto, la más sencilla opción de tratamiento es suspender el
preparado de nitroglicerina y utilizar, si es posible, un medicamento cardíaco alternativo.
Siempre debe considerarse la arteritis temporal como posible causa de cefalea en
pacientes mayores de 50 años. Favorecen este diagnóstico el aumento de la velocidad de
eritrosedimentación; la claudicación mandibular; las artralgias y las mialgias; y una arteria
temporal hipersensible e indurada. El propranolol es un antimigrañoso eficaz. Sin embargo,
antes de prescribir estos fármacos deben descartarse los posibles factores que la
precipitan. La ergotamina es un tratamiento abortivo eficaz para migraña, pero es un
vasoconstrictor y no debe usarse en pacientes con angina de pecho. En la valoración de
ancianos con cefalea de inicio reciente debe considerarse realizar una tomografía
computadorizada del cerebro. En este caso, si es posible suprimir la cefalea al suspender la
nitroglicerina, no es necesaria una CT.

44.- Si en un paciente con síndrome de Cushing encontramos una ACTH muy baja o
suprimida, la primera posibilidad de diagnóstico es:

a) Adenoma suprarreanal.
b) Enfermedad hipotálamo-hipofisiaria.
c) Hiperplasia suprarrenal congénita.
d) Adenoma hipofisiario.

Aproximadamente 20 a 25% de los pacientes con síndrome de Cushing tienen una neoplasia
suprarrenal. En la mayoría de estos casos existe una producción autónoma de cortisol por la
neoplasia, lo que conduce a descenso en los niveles de ACTH a rangos indetectables o
menores a 2 pmol/L o 10 pg/ml, por retroalimentación negativa de la secreción de ACTH.
Williams GH, Dluhy RG. Enfermedades de la corteza suprarrenal. En Jameson JL (ed):
Harrison. Endocrinología. 1a ed. Madrid. MacGraw-Hill España, 2006: 126-127.

45.- ¿La principal causa de hiponatremia intra-hospitalaria es de tipo iatrógeno, y sus


síntomas son?:

a) Irritabilidad, temblor muscular, crisis convulsivas y disminución de los reflejos


tendinosos.
b) Irritabilidad, temblor muscular, crisis convulsivas y aumento de los reflejos
tendinosos.
c) Irritabilidad, debilidad muscular, crisis convulsivas y aumento de los reflejos tendinosos.
d) Astenia, temblor muscular, crisis convulsivas y aumento de los reflejos tendinosos.
La hiponatremia se define como una concentración plasmática de sodio (natremia) inferior a
136 mEq/l. Los síntomas clínicos pueden aparecer con cifras inferiores a 130 mEq/l y se
considera un cuadro grave cuando las cifras son inferiores a 125 mEq/l.

Existen situaciones clínicas en las que al realizar análisis de sangre se detectan valores de
natremia inferiores a la normalidad, que en realidad no son correctas ya que al realizar
determinaciones más precisas la cantidad de sodio es normal, son las falsas hiponatremias o
seudohiponatremias.

Las hiponatremias se pueden agrupar según su mecanismo de producción en:

Pseudohiponatremia inducida por moléculas osmóticamente activas (glucosa, manitol


o glicina). Estas sustancias al llegar al espacio, extracelular provocan un
desplazamiento del agua desde el espacio intracelular sin alterar la cantidad de
sodio, por lo que desciende su concentración (hiponatremia dilucional).
En el caso de la glucosa, un aumento de 100 gr/dl de glucemia provoca un descenso
de 1,7 mEq/l de la natremia. En esta situación la osmolaridad plasmática estará
elevada por la propia glucosa.

Pseudohiponatremia inducida por moléculas no osmóticamente activas (triglicéridos,


proteínas). Estas moléculas reducen el porcentaje relativo de agua de un volumen
determinado de plasma.
La elevación de 1 gr/dl de triglicéridos desciende la natremia aproximadamente 1,7
mEq/l; mientras que la elevación de 1 gr/dl de proteínas plasmáticas causa un
descenso de 1mEq/l de la natremia.
En estos casos, las osmolaridad plasmática estará dentro de los valores de
normalidad.

El cuadro clínico de presentación va a depender de la magnitud de la hiponatremia y de su


velocidad de instauración.

La presencia de sintomatología en el paciente con hiponatremia está en relación con la


hiperhidratación neuronal ocasionada por la entrada de agua en la célula por el descenso de
la osmolaridad en el compartimento extracelular. Síntomas más frecuentes:

Aparato Gastrointestinal: Náuseas, vómitos


Sistema Nervioso Periféric: Calambres musculares, alteraciones visuales
Sistema Nervioso Central: Cefalea, letargia, convulsiones, coma.

En el caso de las hiponatremias agudas, habitualmente debutan con síntomas


gastrointestinales (náuseas y vómitos), aunque se suelen acompañar posteriormente de
calambres musculares y de alteraciones visuales. A medida que descienden las cifras
de sodio, aumenta la gravedad de la sintomatología, hasta que se produce un edema
cerebral que se caracteriza por la presencia de cefalea, náuseas, letargia,
convulsiones y coma.

En el caso de hiponatremias crónicas, el cuadro clínico es mucho más leve para las mismas
cifras de natremia, que en las agudas. Su mayor expresión también es el edema cerebral,
que se produce por la agregación entre pequeños osmolitos (idiosmoles) en el interior de las
neuronas, que favorece el aumento de la osmolaridad intracelular y la entrada de agua en el
interior neuronal.

En las formas crónicas no suelen presentarse síntomas de estupor, convulsiones o coma,


excepto en aquellos casos que presentan cifras de natremia inferiores a 120 mmol/l.

Bibliografía

Al-Salman J, Kemp D, Randall D. Hyponatremia. West J Med. 2002 ;176(3):173-6.


Allison SP, Lobo DN. Fluid and electrolytes in the elderly. Curr Opin Clin Nutr Metab Care.
2004 ;7(1):27-33
Gary G. Singer, Barry M. Brenner. Alteraciones de líquidos y electrolitos. Medicina Interna.
Harrison et al. Ed. McGraw-Hill. 2001. p. 322-336.
Goh KP. Management of hyponatremia. Am Fam Physician. 2004 ;69(10):2387-94.
Halperin ML, Bohn D. Clinical approach to disorders of salt and water balance. Emphasis on
integrative physiology. Crit Care Clin. 2002 ;18(2):249-72. [Medline]
Kugler JP, Hustead T. Hyponatremia and hypernatremia in the elderly. Am Fam Physician.
2000 ;61(12):3623-30 [Texto completo]
Martínez Vea A, Torras Rabasa A. Alteraciones del metabolismo hidrosalino. Medicina
Interna. Farreras-Rozman. Ed. Harcourt. 2000. p. 2073-2086.
Luckey AE, Parsa CJ. Fluid and electrolytes in the aged. Arch Surg. 2003 ;138(10):1055-60
Luque Ramírez M, Bajo Martínez A, Bernal Morell E, Manzano Espinosa L. Hiponatremia
grave asociada al uso de fluoxetina en el anciano. Rev Clin Esp. 2002 ;202(4):246.

46.- Femenino de 32 años tratado actualmente por padecer artritis por fiebre reumática,
la articulación que se afecta con mayor frecuencia en este caso es:

a) El codo.
b) La muñeca.
c) La cadera.
d) La rodilla.

La artritis es generalmente la manifestación inicial de la fiebre reumática, aunque se puede


presentar antes una carditis asintomática. Una o ambas rodillas se afectan en el 76% de
los casos aproximadamente y uno o ambos tobillos en el 50% de los casos.

Gibofsky A, Zabriskie J. Acute rheumatic fever and poststreptococcal arthritis. Harris ED

Jr, Budd RC, Genovese MC, Firestein GS, Sergent JS, Sledge CB, Ruddy S, editors. Kelley’s

Textbook of Rheumatology. Philadelphia: Elsevier Saunders; 2005. p. 1684-96.


47.- Las siguientes son manifestaciones del hipercortisolismo endógeno:

a) Hipertensión arterial y obesidad central.


b) Taquicardia, nerviosismo y temblor.
c) Somnolencia e incremento de peso.
d) Crecimiento de tejidos blandos en zonas acrales e hipertensión.

Fisiopatología de la producción de glucocorticoides

Exceso de cortisol

La producción excesiva de cortisol puede ser el resultado de una sobreproducción de CRH,


de ACTH o tumores adrenales que produzcan cortisol. Todas las causas de
hipercortisolismo endógeno (producidos por las glándulas suprarrenales del paciente) o
exógeno (uso farmacológico) se denominan "síndrome de Cushing". El término "enfermedad
de Cushing" se reserva para los casos en que el origen está en un tumor hipofisiario
productor de ACTH.

Los efectos metabólicos del exceso de glucocorticoides son los siguientes:

1. Aumento de la neoglucogénesis y resistencia a la insulina; esto puede llevar a la


diabetes mellitus.
2. Aumento del catabolismo proteico; esto puede llevar a la emaciación, osteoporosis y
adelgazamiento de la piel.
3. Aumento y redistribución de la grasa corporal: se produce una obesidad de
predominio central, facie de luna, tungo o acúmulo dorsal de grasa, manteniendo
extremidades relativamente delgadas.
4. Involución del tejido linfático y disminución de la respuesta inflamatoria: se
produce una disminución de la inmunidad celular y humoral con lo que aumenta la
susceptibilidad a infecciones.
5. Aumento de la secreción de ácido por el estómago lo que lleva a una predisposición
de úlcera gastroduodenal.
6. Retención de sodio y redistribución de los fluidos corporales lo que produce edema
e hipertensión arterial.
7. Función gonadal: los glucocorticoides afectan la secreción de gonadotrofinas. En los
hombres disminuye la concentración de testosterona. En las mujeres, suprime la
respuesta de LH al GnRH, lo que lleva a una supresión de la secreción de estrógenos
y progestinas, con anovulación y amenorrea.

Todos los efectos anteriormente enunciados pueden ocurrir independientes del origen de
los glucocorticoides. La causa más común de síndrome de Cushing se debe a la
administración exógena de dosis farmacológicas de ellos con fines generalmente
antiinflamatorios e inmunosupresores.
48. - Characteristic to be met by subjects in a cohort study.

a) Having the disease under study.


b) Have a family history of the disease under study.
c) They are adults with multiple illnesses to study.
d) Not having the disease under study.

El estudio de cohorte es el diseño ideal para buscar la causa de una enfermedad por lo que
compara a un grupo de sujetos, libres de la enfermedad en estudio expuestos a los
factores de sospecha con otro grupo de sujetos también libres de la enfermedad, que no
estén expuestos a los factores sospechosos.

Ruiz M. A. Epidemiología Clínica, 1ª. Ed. 2004; Pág.: 279,280

49.- Se trata de paciente de 30 años que refiere datos de ansiedad, enojo e inseguridad
refiriendo que existe una constante sensación de que en su trabajo no realiza
correctamente las funciones que se encomiendan. Esto hace que necesite permanentemente
revisarlas una y otra vez, lo que le supone pérdida de tiempo y eficacia. Esta sensación es
tan dominante en su psiquismo que le conduce a la idea de fracaso y a la pérdida de
autoestima. El diagnóstico inicial corresponde a trastorno:

a) Psicótico paranoide.
b) Por ansiedad fóbica.
c) De personalidad evitativo-fóbica.
d) Obsesivo-compulsivo.

CRITERIOS DEL DSM-IV PARA EL DIAGNÓSTICO DEL TRASTORNO OBSESIVO-


COMPULSIVO. Fuente AMERICAN PSYCHIATRIC ASSOCIATION

Criterios para el diagnóstico de F42.8 Trastorno obsesivo-compulsivo (300.3)

A. Se cumple para las obsesiones y las compulsiones:

Las obsesiones se definen por 1, 2, 3 y 4:

1. pensamientos, impulsos o imágenes recurrentes y persistentes que se


experimentan en algún momento del trastorno como intrusos e inapropiados,
y causan ansiedad o malestar significativos.
2. Los pensamientos, impulsos o imágenes no se reducen a simples
preocupaciones excesivas sobre problemas de la vida real.
3. La persona intenta ignorar o suprimir estos pensamientos, impulsos o
imágenes, o bien intenta neutralizarlos mediante otros pensamientos o
actos
4. La persona reconoce que estos pensamientos, impulsos o imágenes
obsesivos son el producto de su mente (y no vienen impuestos como en la
inserción del pensamiento).

Las compulsiones se definen por 1 y 2:

1. comportamientos (p. ej., lavado de manos, puesta en orden de objetos, comprobaciones)


o actos mentales (p. ej., rezar, contar o repetir palabras en silencio) de carácter repetitivo,
que el individuo se ve obligado a realizar en respuesta a una obsesión o con arreglo a
ciertas reglas que debe seguir estrictamente.
2. El objetivo de estos comportamientos u operaciones mentales es la prevención o
reducción del malestar o la prevención de algún acontecimiento o situación negativos; sin
embargo, estos comportamientos u operaciones mentales o bien no están conectados de
forma realista con aquello que pretenden neutralizar o prevenir o bien resultan claramente
excesivos.

B. En algún momento del curso del trastorno la persona ha reconocido que estas obsesiones
o compulsiones resultan excesivas o irracionales.

Nota: Este punto no es aplicable en los niños.

C. Las obsesiones o compulsiones provocan un malestar clínico significativo, representan


una pérdida de tiempo (suponen más de 1 hora al día) o interfieren marcadamente con la
rutina diaria del individuo, sus relaciones laborales (o académicas) o su vida social.

D. Si hay otro trastorno, el contenido de las obsesiones o compulsiones no se limita a él (p.


ej., preocupaciones por la comida en un trastorno alimentario, arranque de cabellos en la
tricotilomanía, inquietud por la propia apariencia en el trastorno dismórfico corporal,
preocupación por las drogas en un trastorno por consumo de sustancias, preocupación por
estar padeciendo una grave enfermedad en la hipocondría, preocupación por las
necesidades o fantasías sexuales en una parafilia o sentimientos repetitivos de culpabilidad
en el trastorno depresivo mayor).

E. El trastorno no se debe a los efectos fisiológicos directos de una sustancia (p. ej.,
drogas, fármacos) o de una enfermedad médica.

Especificar si:

Con poca conciencia de enfermedad: si, durante la mayor parte del tiempo del episodio
actual, el individuo no reconoce que las obsesiones o compulsiones son excesivas o
irracionales.
50.- Menciona cuál es la característica principal de los estudios experimentales que los
distingue de los observacionales:

a) Los grupos de estudio y de control son del mismo tamaño.


b) El investigador determina cuales sujetos recibirán la exposición.
c) Los grupos de estudio y de control son compatibles y tienen las mismas características.
d) Se usan controles.

Aunque existen múltiples clasificaciones de los diseños de investigación, la mayoría de los


autores están de acuerdo en la existencia de dos grandes tipos: experimentales y
observacionales (no experimentales). La diferencia básica estriba en el método empleado
para asignar a los sujetos del estudio a la exposición o intervención.

Ruiz M. A. Epidemiología Clínica, 1ª. ED. 2004; Pág.: 234

51.- Un varón de 45 años de edad con cirrosis hepática tuvo dolor abdominal generalizado
durante 24 h sin náuseas ni vómitos. Su temperatura es de 38.3°C y ha tenido distensión
abdominal con onda de líquido claro. Hay hipersensibilidad difusa en la palpación abdominal.
En la paracentesis se obtuvo líquido transparente con 816 leucocitos/mm3 (85%
polimorfonucleares, 15% linfocitos). La tinción de Gram no muestra bacterias. ¿Cuál de los
siguientes diagnósticos es más probable?

a) Enfermedad ulcerosa péptica.


b) Peritonitis primaria.
c) Pancreatitis.
d) Colecistitis.

Allen R. M. MMS Medicina Interna. 5ª. Edición. National Medical Series. Mc. Graw Hill.
2006. (capítulo 8 V E 1 a). El diagnóstico más probable es peritonitis primaria. Aunque es
difícil diferenciar la peritonitis primaria (espontánea) por rotura de víscera hueca y
contaminación peritoneal, la presencia de fiebre y la leucocitosis la ascitis sugiere alguna
clase de infección peritoneal. La pancreatitis se caracteriza por dolor localizado intenso
(mesoepigástrico), que se irradia a la espalda. En general, las náuseas y vómitos no se
relacionan con pancreatitis aguda. El dolor abdominal en caso de colecistitis se sitúa en el
cuadrante superior derecho y suele haber náuseas y vómitos. El absceso hepático tiende a
ser un trastorno subagudo sin datos peritoneales prominentes. En caso de ascitis crónica
infectada, las enfermedades ulcerosas pépticas son causa poco probable.

52.- Ante un paciente con datos de sugestivos de embolia pulmonar, el diagnóstico


definitivo se realiza mejor con el uso de:

a) Gasometría arterial.
b) Radiografía torácica .
c) Electrocardiograma (ECG) .
d) Arteriografía pulmonar.

Allen R. M. MMS Medicina Interna. 5ª. Edición. National Medical Series. Mc. Graw Hill.
2006. (capítulo 2 VIII E 6 a-c, 7, 8 a). La prueba más sensible y específica de embolia
pulmonar es la arteriografía pulmonar. El gammagrama nuclear pulmonar es otra técnica útil,
pero no es tan específico como la arteriografía. Aunque un gammagrama normal casi
descarta embolia pulmonar, los resultados a menudo caen en el límite de probabilidad inter-
medio, lo que dificulta establecer un diagnóstico definitivo. En la mayor parte de los casos
de embolias pulmonares, el electrocardiograma (ECG) es normal. La desviación aguda del eje
a la derecha observado en ECG puede producir diagnóstico erróneo de infarto miocárdico
anterior (MI). La hipoxia, la hipocapnia y la alcalosis respiratoria son datos clásicos de la
gasometría, pero son inespecíficos de embolia pulmonar. La radiografía de tórax es normal,
sobre todo si no ha habido infarto.

53.- Son elementos fundamentales del diseño metodológico en un ensayo clínico controlado:

a) La asignación es aleatoria, requiere la intervención directa del investigador,


necesariamente comparativo con control estricto de las variables, utiliza la
maniobra ciega y proviene de una hipótesis matemática.
b) Retrospectivos, de observación y comparativos. Parten del efecto a la causa y son
débiles por los sesgos de selección.
c) Bloques de pacientes evaluados desde antes de presentar el efecto, requieren de
una maniobra doble ciego, solo pueden medirse con estadística descriptiva y poseen
un alto valor en el estudio de enfermedades muy frecuentes.
d) Solo utilizan una serie de casos.

Córdova VH, Jiménez J, Jiménez MC. Manual de diseño metodológico en investigación


clínica. ULSA – UAPY 2001. Pág. 30 – 31. El ensayo clínico controlado es un diseño muy
cercano a un experimento, solo que en seres humanos. Busca respuestas concretas a
cuestionamientos bien delimitados, utilizando intervenciones directas sobre la
población estudiada, relacionados con la curación o el control de una medida terapéutica,
para lograrlo utiliza algunos elementos fundamentales, como: la asignación aleatoria,
maniobras doble “ciego”, la intervención por parte del investigador, la comparación
entre los grupos, procedimientos de seguimiento muy rigurosos, así como, el
consentimiento informado dentro de márgenes bioético.

54.- Se trata de paciente femenino de 43 años de edad con diagnóstico de polimiosistis al


realizar la exploración física, la disminución de la fuerza muscular se caracteriza por:

a) Ser de predominio proximal.


b) Ser de predominio distal.
c) Acompañarse de dolor intenso.
d) Afectar únicamente a la musculatura distal.

La polimiositis (PD) y dematomiositis (DM), son enfermedades inflamatorias en las cuales el


compromiso principal es la debilidad muscular, generalmente proximal y simétrica con
atrofia subsecuente, generalmente indolora.

Su etiología y patogenia siguen siendo desconocidas, siendo considerada dentro de las


enfermedades difusas del tejido conectivo.

Numerosos estudios sugieren que una exposición previa a diversos virus como Influenzae A
y B, Coxsackie virus tipo B y Picornavirus pueden desencadenar PM-DM; además se cree
que los fenómenos inmunológicos juegan un papel preponderante, por la presencia de
autoanticuerpos, depósito de inmunoglobulinas y complemento en las paredes vasculares.

A nivel anatomo-patológico las principales alteraciones se encuentran en los músculos


esqueléticos y en los vasos sanguíneos. Siendo la vasculitis un signo común en la
dermatomiositis infantil y su severidad indicará una peor evolución. Clínicamente los
síntomas iniciales son la debilidad muscular proximal, el rash típico, frecuente el edema en
cara y a veces en miembros, fiebre moderada, posteriormente dificultad para la deglución y
debilidad en la voz; asimismo manifestaciones viscerales que no se presentan siempre, pero
son de gran importancia en cuanto al pronóstico, asimismo trastornos ventilatorios
(pulmonares): por afectación primaria (neumonía intersticial) o secundaria a la disfunción
faríngea (aspiración) o a la debilidad de los músculos respiratorios (insuficiencia
ventilatoria), o como complicación del tratamiento (infecciones oportunistas).

Wortmann RL. Idiopathic inflammatory myopathies. A. Clinical features. In: Primer on the
rheumatic diseases. New York: Springer-Arthritis Foundation; 2008. p. 363-7.
55.- Acude al servicio de urgencias masculino de 34 años acompañado por su esposa agitado
e inquieto porque dice que “le persiguen unos asesinos que van a matarle”. Se realiza
exploración física observándose pupilas midriáticas, temperatura de 37,9ºC, Fc 110 lpm y
TA de 155/95 mmHg, sin otros patológicos aparentes. Su familiar afirma que tiene historia
de abuso de drogas. La droga que más probablemente ha producido esta reacción es:

a) Heroína.
b) Cocaína.
c) Diacepam.
d) Alcohol.

Los criterios internacionales de diagnóstico de acuerdo al


DSM-IV TR son los que a continuación se detallan:

A. Consumo reciente de cocaína.

B. Cambios psicológicos comportamentales desadaptativos clínicamente significativos


(sexualidad inapropiada, comportamiento agresivo, labilidad emocional, deterioro de la
capacidad de juicio y deterioro de la actividad laboral o social) que se presentan durante la
intoxicación o pocos minutos después del consumo de cocaína.

C. Dos o más de los siguientes signos, que aparecen durante o poco tiempo después del
consumo de cocaína:

(1) Taquicardia o bradicardia.


(2) Dilatación pupilar.
(3) Aumento o disminución de la tensión arterial.
(4) Sudoración o escalofríos.
(5) Náuseas o vómitos.
(6) Pérdida de peso demostrable.
(7) Agitación o retraso psicomotores.
(8) Debilidad muscular, depresión respiratoria, dolor en el pecho o arritmias cardíacas.
(9) Confusión, crisis comiciales, discinesias, distonías o coma.
D. Los síntomas no se deben a enfermedad médica ni se explican mejor por la presencia de
otro trastorno mental.

La característica esencial de intoxicación por cocaína es la presencia de cambios


psicológicos o comportamentales desadaptativos clínicamente significativos que aparecen
durante o poco tiempo después del consumo de cocaína (Criterios A y B). La intoxicación
por cocaína empieza habitualmente con una sensación de euforia (high) e incluye uno o más
de los síntomas siguientes: euforia con incremento de la sensación de vigor, sociabilidad,
hiperactividad, inquietud, hipervigilancia, sensibilidad interpersonal, charlatanería,
ansiedad, tensión, estado de alerta, grandiosidad, comportamientos estereotipados y
repetitivos, rabia o cólera y deterioro de la capacidad de juicio y, en el caso de intoxicación
crónica, afectividad embotada, cansancio o tristeza y retraimiento social. Estos cambios
psicológicos y comportamentales se acompañan de dos o más de los siguientes signos y
síntomas, que aparecen durante la intoxicación o poco tiempo después: taquicardia o
bradicardia, dilatación pupilar, aumento o disminución de la tensión arterial, sudoración o
escalofríos, náuseas o vómitos, pérdida de peso demostrable, agitación o retraso
psicomotores, debilidad muscular, depresión respiratoria, dolor en el pecho o arritmias
cardíacas y confusión, crisis comiciales, discinesias, distonías o coma (Criterio C). La
intoxicación aguda o crónica se asocia a menudo con deterioro de la actividad social o
laboral. La intoxicación grave puede conducir al coma. Para establecer el diagnóstico de
intoxicación por cocaína los síntomas no han de ser debidos a enfermedad médica ni
explicarse mejor por la presencia de otro trastorno mental (Criterio D).
La magnitud y el tipo de los cambios psicológicos o comportamentales dependen de muchas
variables, que incluyen la dosis consumida y las características individuales del sujeto que
consume la sustancia (p.ej., tolerancia, grado de absorción, cronicidad del consumo y
contexto en el que se ingiere la droga). Los efectos estimulantes observados más
frecuentemente son euforia, aumento del pulso y la tensión arterial, y actividad
psicomotora. Los efectos depresores como tristeza, bradicardia, descenso de la tensión
arterial y disminución de la actividad psicomotora son menos frecuentes y sólo aparecen
con el consumo crónico de dosis altas.

56.- El signo clínico que indica irritación del nervio ciático como componente de una
lumbalgia se llama:

a) Signo de Lassegue.
b) Signo de Galeazzi.
c) Signo de McMurray.
d) Signo de Filkestein.
La lumbalgia es una patología muy frecuente en nuestro medio, con una enorme
repercusión sanitaria y sociolaboral. Con la edad, se inicia una cascada degenerativa en la
columna lumbar, que comienza en el disco intervertebral, continuando por las facetas
articulares y demás elementos vertebrales. Esta degeneración del raquis forma parte del
envejecimiento normal del individuo, aunque en ocasiones puede causar dolor y/o
alteraciones neurológicas. Para comprender la fisiopatología del dolor lumbar, será
preciso conocer que la inervación de la columna lumbar se hace fundamentalmente por
tres ramos nerviosos: ramo dorsal de los nervios espinales lumbares (o rami dorsal),
nervio sinuvertebral de Luschka y ramos ventrales de la cadena simpática. Existen dos
tipos de patrones de dolor en la columna lumbar: el dolor irradiado y el dolor referido. La
cascada de la degeneración consta en tres estadios. El primer estadio sería la disfunción.
El anillo fibroso se fisura y pierde la capacidad de contener al núcleo pulposo. Esto
ocasiona primero el síndrome de disrupción discal y, si el núcleo supera el contorno del
annulus, las hernias discales. El segundo estadio de Kirkaldy-Willis es el de inestabilidad.
En este estadio la movilidad en el segmento móvil aumenta de forma patológica.

Maniobra de Lassègue: Es la más importante, es casi patognomónica.

Con el paciente en decúbito supino se levanta la pierna extendida. Se considera positiva

cuando aparece un dolor intenso a nivel lumbar, en la pierna o en ambos niveles, entre los

30º-75º de flexión en la cadera, debido al estiramiento del nervio ciático. Pasados los 70º

puede aparecer un falso positivo (dolor también en sujetos sanos) por la distensión de los

isquiotibales. Para descartarlo realizaremos otras maniobras diagnósticas.

1. Andersson GBJ. Epidemiologic features of chronic low-back pain. Lancet. 1999;354:5815.


2. Estudio EPISER. Sociedad Española de Reumatología. 2000.
3. Herrera Rodríguez A, Rodríguez Vela J. Estenosis de canal lumbar. Rev Ortop
Traumatol. 2002;4:351-72.
4. MacGregor AJ, Andrew T, Sambrook PN, Spector TD. Structural, psychological, and
genetic influences on low back and neck pain: a study of adult female twins. Arthritis
Rheum. 2004;51:160-7.
5. Kim KS, Yoon ST, Park JS, Li J, Park MS, Hutton WC. Inhibition of proteoglycan and
type II collagen synthesis of disc nucleus cells by nicotine. J Neurosurg Spine.
2003;99:291-7.
6. Fujiwara A, Tamai K, Yamato M, An HS, Yoshida H, Saotome K, et al. The relationship
between facet joint osteoarthritis and disc degeneration of the lumbar spine: an MRI

study. Eur Spine J. 1999;8:396-401.


7. Eyre DR, Muir H. Types I and II collagens in intervertebral disc. Interchanging
radialdistributions in annulus fibrosus. Biochem J. 1976;157:267-70.56.-5
57.- El clásico inicio del dolor por apendicitis es en:

a) Hipogastrio.
b) Fosa lumbar derecha.
c) Epigastrio.
d) Fosa iliaca derecha.

Cuadro clínico y diagnóstico.

El cuadro clínico suele instaurarse en pocas horas. La secuencia clásica de los síntomas es la
siguiente: 1º dolor abdominal en epigastrio;2º náuseas, vómitos;
3º cambio del dolor hacia fosa ilíaca derecha y aparición de hiperestesia a la presión
profunda; 4º fiebre 38.3-39.4ºC; 5º leucocitosis (10000-20000/dL). Cuando el orden de
aparición de estos síntomas es diferente debe reconsiderarse el diagnóstico. El dolor es el
síntoma más constante. En forma característica el dolor aparece por la noche, en general
en el epigastrio o la región peri umbilical; otras veces es difuso en todo el abdomen o, con
menor frecuencia, se localiza directamente en la fosa ilíaca derecha. Es un dolor continuo,
de intensidad moderada, que aumenta con la tos, los movimientos respiratorios o la
deambulación y no cede tras el vómito. Puede tener exacerbaciones cólicas y obliga a
guardar cama. A las pocas horas se localiza definitivamente en la fosa ilíaca derecha y en
algunos pocos casos puede presentar irradiación testicular. El origen del dolor es la
distensión de la luz apendicular. Una mejoría brusca del dolor suele indicar perforación del
órgano. Al palpar el abdomen se aprecia dolor a la presión profunda y puede observarse ya
rigidez refleja de la pared abdominal en la fosa ilíaca derecha. No obstante, la aparición de
rigidez parietal, inconstante, demuestra ya la existencia de irritación del peritoneo parietal
(peritonitis localizada), con lesiones apendiculares avanzadas.

Ferrada R.: Guías para el manejo de urgencias- Apendicitis aguda. Hospital universitario
Evaristo García. 1996.

Guss D.: Comparison of men and women presenting with acute appendicitis. Am. J. Emer.
Med. 2000. 18: 4: 372-376.
58.- ¿Cuál es la principal complicación pulmonar postquirúrgica?

a) Neumonía.
b) Atelectasias.
c) Derrame pleural.
d) Neumotórax.

Bruce E. Jarrell, R. Anthony Carabasi, Nacional Medical Series for Independent Study.
Wiliams & Wilkins, 3rd Edition: 27-50.

Las atelectasias por mucho son la complicación más frecuente de los pacientes posoperados
la cual se trata con medidas generales como inspirometría incentiva y fisioterapia pulmonar.
La complicación pulmonar que le continúa en frecuencia son los procesos neumónicos.

59.- La localización más frecuente del quiste hidatídico en el hombre es:

a) Hueso.
b) Hígado.
c) Pulmón.
d) Cerebro.

INTRODUCCIÓN
La hidatidosis humana es una zoonosis parasitaria, causada por la forma quística que
adquiere el hombre, así como también otros huéspedes intermediarios, por la ingestión de
huevos del Echinococcus granulosus, los mismos se adquieren por alimentos contaminados a
partir de las heces del perro, produciendo una parasitación hepática, pulmonar o en otros
órganos.

Esta enfermedad aparentemente benigna debe ser considerada grave, no solo por las
complicaciones evolutivas a que está expuesto y que pueden ser mortales, sino por la
compleja terapéutica que puede requerir y la elevada morbi-mortalidad que en algunas
series alcanzan el 10%. Pueden desarrollarse como quiste hidatídico único en el 90%, dos ó
múltiples quistes (3-4%) y las infestaciones masivas son raras. (2)

Localización

Las hidatidosis humana prevalece en el hígado (50-60%), le sigue en frecuencia el pulmón


(20-30%), las otras localizaciones son peritoneo (3-5%), bazo (1,5-3%), Riñón (1.5-2%), Sist.
Nervioso (2%), huesos (2%), corazón (0,2%), piel y músculos (4%).

En el hígado puede ocupar cualquier lugar, pero la ubicación más frecuente es en el lóbulo
derecho, y con mayor frecuencia lo hace en la parte posterosuperior de este (segmentos
hepáticos VII y VIII de la clasificación de Couinaud). (6)
1. Farreras P, Rozman C. Medicina Interna. 13ra. ed. Madrid. España: Harcourt Brace;
1995: 2: 2472-2474.
2. Correa Tineo S, Culqui Lévano C, Pinto Paz M, Huillca L ,Salinas Alva E. Hidatidosis
hepática: Revisión de casos intervenidos quirúrgicamente en el Hospital Militar Central.
Rev. de Gastroent del Perú, 2000:20(3):270-284.
3. Larrieu E, Frider B, del Carpio M y col. Portadores asintomáticos de hidatidosis:
epidemiología, diagnóstico y tratamiento. Rev Panam Sal Pub/Pan Am J Public
Health,2000: 8(4):250-256.
4. Merk HB y col. El Manual Merk: de diagnóstico y tratamiento. 10ma ed. Madrid (España):
Ediciones Harcourt; 2001: Sec13, Cap161.
5. Vera MG, Venturelli F, Aliro Venturelli A. Hidatidosis humana. Cuad de Cirug (Valdivia),
2003, 17:88-94
6. Feraina P, Oria A. Cirugía de Michans. 5ta ed. Buenos Aires: El ateneo; 2003: 519-528
7. Atias A, Neghme A. Parasitología Clínica. 2da. ed. Santiago de Chile: Publicaciones
Técnicas Mediterráneo Ltda, 1984: 309-321
8. Harrison. Principios de le Medicina interna 14ta ed. Madrid (España): Mc Graw-Hill-
Interamericana, 1998: vol 1: 1404-1405
9. Galati G, Sterpetti AV, Caputo M. Endoscopic retrograde cholangiography for
intrabiliary rupture of hydatid cyst. Am J Surg, 2006: 191(2):206-210.

60. - Una mujer de 17 años tiene exantema cutáneo rojo difuso; fiebre de 39.4°C y diarrea
leve acuosa. En fechas recientes tuvo infección de garganta por la que se le administró
sulfametoxazol. Comenzó sus menstruaciones hace tres días. En la exploración física se
encuentran cambios eritematosos difusos de la piel con descamación temprana. La boca y
las conjuntivas están eritematosas. ¿Cuál de los siguientes explica todo el proceso?

a) Bacteriemia por Salmonella.


b) Síndrome de choque tóxico (TSS).
c) Tuberculosis.
d) Mononucleosis por virus de Epstein-Barr.

Allen R. M. MMS Medicina Interna. 5ª. Edición. National Medical Series. Mc. Graw Hill.
2006. (Capítulo 8 VII D 1 a, 2 b). La bacteriemia por Salmonella, el síndrome de choque
tóxico (TSS), la tuberculosis y la mononucleosis de Epstein-Barr pueden acompañarse de
fiebre, pero la presencia de exantema descamativo difuso sugiere TSS, reacción
farmacológica grave (p. ej., síndrome de Stevens-John-son); enfermedad de Kawasaki, o
escarlatina. El exantema cutáneo relacionado con salmonelosis es muy sutil y evanescente
(manchas de color rosa). La tuberculosis no se caracteriza por afección cutánea difusa y de
mucosas o diarrea acuosa. Si bien la alergia al sulfametoxazol puede producir eritema
cutáneo y de mucosas, no causa diarrea.
61.- ¿Cuáles son los actos obsesivos más frecuentes en el trastorno obsesivo-compulsivo?:

a) Recuentos mentales.
b) Evitar pisar las cruces de las baldosas.
c) Comprobaciones y rituales de limpieza.
d) Acumulación y colección de objetos.

Síntomas del TOC

Obsesiones

Las obsesiones son ideas o impulsos no deseados que aparecen repetidamente en la mente
de la persona que padece TOC. Los pacientes suelen tener miedo a sufrir daño ellos mismos,
o alguien al que quieren, se preocupan irracionalmente por no contaminarse, o tienen una
necesidad excesiva de hacer las cosas correctamente o con perfección. Una y otra vez, la
persona piensa algo que le inquieta, como por ejemplo, "Mis manos pueden estar
contaminadas--debo lavarlas" o "Puedo haber dejado el gas abierto" o "Estoy haciéndole
daño a mi hijo." Estos pensamientos angustiosos se inmiscuyen con los otros pensamientos
del paciente, y causan ansiedad. A veces, las obsesiones son de carácter violento o sexual, o
tienen que ver con enfermedades.

Compulsiones

En respuesta a sus obsesiones, la mayoría de las personas con TOC recurren a


comportamientos repetitivos llamados compulsiones. Los más frecuentes son los rituales de
limpieza y las comprobaciones. Otros comportamientos compulsivos incluyen recuentos (a
menudo al mismo tiempo que se realizan otras acciones compulsivas, tales como lavarse las
manos), hacer repeticiones, acaparamiento, y recolocaciones sin fin de objetos en un
esfuerzo para mantenerlos perfectamente alineados. También son corrientes los problemas
mentales, tales como repetir frases mentalmente y hacer listas. Estos comportamientos, en
general, tienen por objeto proteger de peligros a la persona que padece TOC, o a los otros.

Algunas personas con TOC tienen rituales establecidos; otros tienen rituales que son
complejos y cambiantes.

El ejecutar estos rituales sólo proporciona alivio temporal de la ansiedad, pero no hacerlos
incrementa la ansiedad de la persona.
62.- El siguiente es el síntoma más específico de la depresión mayor:

a) Palpitaciones.
b) Anorexia.
c) Mejoría vespertina.
d) Alivio tras el llanto.

Trastorno Depresivo Mayor o Depresión Mayor

Se caracteriza por la presencia de uno o más Episodios Depresivos Mayores. El Episodio


Depresivo Mayor abarca 5 o más de los siguientes síntomas:

• Estado de ánimo depresivo la mayor parte del día, casi todos los días según lo indica
el sujeto (por ej. se siente triste o vacío) o la observación realizada por otros (por
ej. llanto).
• Disminución importante del interés o de la capacidad para el placer (disfrutar la
vida) en todas o casi todas las actividades
• Pérdida importante de peso (sin haber hecho régimen) o aumento significativo de
peso (por ej. un cambio de más del 5% del peso corporal en un mes), o disminución o
aumento del apetito casi cada día.
• Insomnio o hipersomnia (sueño durante el día).
• Agitación o enlentecimiento psicomotores.
• Fatiga o pérdida de la energía.
• Sentimientos de inutilidad o de culpa excesiva o inapropiada (no simplemente los
autorreproches o culpabilidad por el hecho de estar enfermo).
• Disminución de la autoestima y de la confianza en si mismo.
• Disminución de la capacidad para pensar, tomar decisiones o concentrarse.
• Visión pesimista, poco promisoria o "negra" del futuro.
• Pensamientos recurrentes de muerte (no sólo temor a la muerte), ideación suicida
recurrente sin un plan específico o una tentativa de suicidio o un plan específico
para suicidarse.

Estos síntomas persisten por lo menos dos semanas y producen un malestar muy
significativo que altera la vida familiar, social, laboral y de otras áreas importantes de la
actividad de la persona. Cuando estos episodios depresivos mayores se repiten (Depresión
Mayor Recurrente) suele haber por lo menos un intervalo libre síntomas, de dos meses,
entre uno y otro.

En la forma Melancólica de la depresión mayor se incluyen como síntomas típicos:

• Falta de reactividad a los estímulos habitualmente placenteros (no se siente mejor,


ni siquiera temporalmente, cuando sucede algo bueno).
• Una cualidad distinta del estado de ánimo depresivo (por ej. la depresión se
experimenta en forma diferente al tipo de sentimiento que acompaña a la muerte
de un ser querido).
• La depresión es habitualmente mayor por la mañana, mejorando el estado de ánimo
a la tarde noche.
• Despertar precoz (por ej. 2 horas antes de lo habitual) que suele estar acompañado
de angustia.

Referencias Bibliográficas

- Diagnostic and Statistical Manual of Mental Disorders, 4° edition (DSM-IV TR).


American Psychiatric Association. American Psychiatric Press, 2000.
- Kaplan and Sadock's Synopsis of Psychiatry, 9° edition. Lippincott Williams & Wilkins
Press, 2003.

63.- Masculino de 50 años con dolor precordial relacionado al esfuerzo, de corta duración,
de 4 meses de evolución y con ECG en reposo normal, se indica:

a) Prueba de esfuerzo con protocolo de Bruce.


b) Ecocardiograma de reposo.
c) Prueba de Talio- Dipiridamol.
d) Ecocardiograma con estrés farmacológico.

El propósito fundamental de la prueba es el de demostrar la existencia de isquemia


miocárdica en los subgrupos de población con mayor prevalencia de cardiopatía isquémica o
bien en los subgrupos en donde la prueba se efectúa a manera de evaluación del
tratamiento en pacientes ya conocidos con cardiopatía isquémica. Existe otro subgrupo de
población en quienes este estudio ayuda a determinar la clase funcional en la que se
encuentran e incluye a pacientes con valvulopatías o con insuficiencia cardiaca de cualquier
etiología.

Protocolos de esfuerzo.

El protocolo más empleado es el de Bruce sobre treadmill, aunque existen otros protocolos
y su elección dependerá de las condiciones del individuo.

Los protocolos discontinuos son los que alternan periodos de esfuerzo que se intercalan con
periodos de reposo de duración similar, se emplean en escasas circunstancias. Los
protocolos continuos son los que no interrumpen el esfuerzo una vez iniciado hasta
finalizada la prueba, permiten mejor adaptación física y psicológica y es posible adaptar la
intensidad de forma individualizada para que la prueba tenga una duración de 6 a 12
minutos.

Los protocolos máximos son los que se suspenden debido a la sintomatología del paciente, a
los signos registrados durante la prueba o se alcanzan valores máximos de FC y VO2. Los
protocolos submáximos son los que se suspenden cuando el sujeto alcanza un nivel
determinado de carga, habitualmente el 85% de la FC máxima teórica (que se encuentra
entorno a los 170 lpm).

En la práctica diaria, el nivel de carga (VO2) se expresa en forma de trabajo externo (MET
ó equivalentes metabólicos) que corresponden a 3,5ml/kg/min de VO2, lo que permite
comparar protocolos entre sí (cada protocolo dispone de fórmulas para realizar el cálculo
de los METS), el error que cometen en el cálculo de los METS es mayor en protocolos
discontinuos.

Emplear la FC como único criterio para determinar el esfuerzo máximo es erróneo, por lo
que deberían tenerse en cuenta otros criterios, como es la percepción subjetiva por parte
del paciente mediante la escala de Borg (tabla I). Esta dificultad en la predicción del
esfuerzo máximo es lo que limita la realización de pruebas submáximas a la determinación
de la condición física de sujetos aparentemente sanos.

Tabla I. Escala de Percepción del esfuerzo de Borg.


Escala de 15 grados Escala de 10 grados
Valor Percepción Valor Percepción
6 No se siente nada 0 Nada
7 0,5 Muy muy leve
Muy muy leve
8 1 Muy leve
9
Muy leve 2 Leve
10
11 Considerablemente leve 3 Moderada
12 4 Algo fuerte
Moderadamente dura
13 5 Fuerte o intensa
14 6
Dura Muy fuerte
15 7
16 8
Muy dura
17 9 Muy muy fuerte
18 (submáxima)
Muy muy dura 10
19
20 Esfuerzo máximo
* A la izquierda la escala original de esfuerzo percibido en 15 grados (de 6 a 20) y a
la derecha la más nueva de 10 categorías.

Tabla II. Indicaciones clásicas de la ergometría.


I. Fines diagnósticos
A. Pacientes sintomáticos.
1. Dolor torácico:
a) Típico
b) Atípico
2. Clínica de equivalentes isquémicos.
A. Pacientes asintomáticos.
1. Con alteraciones en el ECG sugestivas de isquemia.
2. Con alta probabilidad de padecer Cardiopatía Isquémica (paciente con
múltiples factores de riesgo)
3. Cuando convenga descartar con cierta seguridad CI.
4. Con sospecha de CI silente.
5. Sedentarios que inician programa de actividad física.
6. Para estudio funcional de ciertas arritmias.
II. Con fines valorativos y pronósticos.
1. Seguimiento de paciente con CI conocida.
2. Tras IAM.
3. En exámenes prelaborales o laborales.
4. De la eficacia del tratamiento:
Médico.
Cateterismo y angioplastia.
Quirúrgico.
5. Respuesta de la Tensión Arterial.
6. En valvulopatías o miocardiopatías.
7. Estudio de arritmias y trastornos de la conducción aurículo-ventricular.
8. En cardiopatías congénitas.
Bibliografía

1. Gibbons RJ (Edit.). ACC/AHA 2002 Guideline Update of Exercise Testing. 2002


American College of Cardiology Foundation and American Heart Association
ACC/AHA; 2002 [Acceso 1-4-06]. Disponible en:
2. Guidelines for cardiac exercise testing. ESC Working Group on Exercise Physiology,
Physiopathology and Electrocardiography Eur Heart J 1993; 14: 969-988.
3. Fernando Arós Aros F, Boraita A, Alegria E, Alonso AM, Bardaji A, Lamiel R el al.
Guías de práctica clínica de la Sociedad Española de Cardiología en pruebas de
esfuerzo. Rev Esp Cardiol 2000; 53 (8): 1063-94
4. Chaitman B. Las pruebas de esfuerzo. En: Braunwald E, editor. Tratado de
Cardiología. Medicina Cardiovascular. 4ª ed. Madrid Mc-Graw-Hill-Interamericana de
España; 1993. p. 177-197.
5. Schlant RC, Friesinger GC 2nd, Leonard JJ. Clinical competence in exercise testing:
a statement for physicians from the ACP/ACC/AHA Task Force on Clinical Privileges
in Cardiology. J Am Coll Cardiol 1990; 16: 1061-5
6. Reyes Lopez de los M, Iñiguez Romo A, Goicolea de Oro A, Funes Lopez B, Castro
Beiras A. El consentimiento informado en cardiología. Rev Esp Cardiol 1998; 51: 782-
796.
7. Fletcher GF, Flipse T, Malouf J, Kligfield P. Current status of ECG stress testing.
Curr Probl Cardiol. 1998 Jul; 23(7): 353-423.
8. Alegría Ezquerra E, Alijarde Guimerá M, Cordo Mollar JC, Chorro Gascó FJ,
Pajarón López A. Utilidad de la prueba de esfuerzo y de otros métodos basados en el
electrocardiograma en la cardiopatía isquémica crónica. Rev Esp Cardiol 1997; 50: 6-14
9. Wasserman K, Hansen JE, Sue DY, Whipp BJ, Casaburi R. Principles of exercise
testing and interpretation . 2ª ed. Philadelphia: Lea & Febiger; 1994. p. 95-111.
10. American college of Sports Medicine. Guideliness for exercise testing and
prescription. 5ª ed. Baltimore: Williams & Wilkins; 1995.
11. Borg GA. Psychophysical bases of perceived exertion. Med Sci Sports Exerc 1982;
14: 377-381
12. Froelicher VF, Umann TM. Exercise testing: clinical applications. En: Pollock ML,
Schmidt DH, editors. Heart disease and rehabilitation . 3ª ed. Champaign, IL: Human
Kinetics, 1995; p.57-79.
13. Myers J, Froelicher VF. Exercise testing. Procedures and implementation. Cardiol
Clin. 1993; 11(2): 199-213.
14. Weiner DA, McCabe C, Hueter DC, Ryan TJ, Hood WB Jr. The predictive value of
anginal chest pain as an indicator of coronary disease during exercise testing. Am
Heart J 1978; 96: 458-462.

64.- La secreción excesiva de moco traqueobronquial suficiente para producir tos y


expectoración durante 3 meses como mínimo en 2 años consecutivos, se traduce como:

a) Bronquitis crónica.
b) Enfisema.
c) Asma.
d) Asbestosis.

Bronquitis crónica en toda persona que tenga tos y expectoración persistente durante tres
meses al año durante dos años consecutivos como mínimo. En la bronquitis crónica simple,
los pacientes no tienen signos de obstrucción al paso del aire. Algunos de estos pacientes
pueden tener unas vías respiratorias hiperreactivas, que producen broncoespasmo y
sibilancias de forma intermitente. Es lo que se llama bronquitis asmática crónica.

BIBLIOGRAFÍA
1. Guías para el diagnóstico y el tratamiento de la Enfermedad Pulmonar Obstructiva
crónica
Edición Especial 2003 INER
2. Pawles RA. Global Strategy for the diagnosis, management and prevention of chronic
obstructive pulmonary disease. Initiative for Chronic Obstructive Lung Disease (GOLD).
Am
J Respir Crit Care Med 163. 2001;1256-1276.
3. Fabri LM. Global strategy for the diagnosis, management and prevention of COPD;2003
Update. Eur Respir S. 2003;22:
4. Sin, Don D. Contemporary Management of Chronic Obstructive Pulmonary Disease:
Scientific Review. JAMA . 2003;290:2301-2312.
5. Man S. Contemporary Management of Chronic Obstructive Pulmonary Disease: Clinical
Applications. JAMA.2003;290:2313-2316.
6. Sutherland ER. Current Concepts Management of Chronic Obstructive Pulmonary
Disease.
N Engl J Med.2004;350:2689-2697
7. Hurst JR. Chronic obstructive pulmonary disease: the clinical management of an acute
exacerbation. Postgrad Med J;80:497-505
65.- Masculino de 36 años con poliuria se le practica una prueba de deshidratación.
Después de la restricción de líquidos, su osmolalidad urinaria máxima es de 550 mosm/kg y
la plasmática es de 295 mosm/kg. Luego de 1 h de aplicar la inyección subcutánea de 5 U de
vasopresina acuosa, la osmolalidad urinaria es de 860 mosm/kg. ¿Cuál de los siguientes
diagnósticos es probable?

a) Sano.
b) Diabetes mellitus.
c) Diabetes insípida parcial.
d) Diabetes insípida nefrógena.

Allen R. M. MMS Medicina Interna. 5ª. Edición. National Medical Series. Mc. Graw Hill.
2006. (capítulo 9 I B 1 c; cuadro 9-2). El paciente tiene diabetes insípida parcial. La
respuesta definitiva del enfermo a la inyección de hormona antidiurética (ADH) indica que
no produce concentraciones máximas eficaces de ADH después de la restricción de líquidos,
y por tanto tiene diabetes insípida parcial o completa. La capacidad para lograr una
concentración urinaria normal o casi normal indica que el déficit de ADH sólo es parcial. La
respuesta de ADH descarta diabetes insípida nefrógena. La diabetes mellitus, otra causa
de poliuria, se diagnostica por las concentraciones de glucosa en orina y sangre más que por
estudios del procesamiento renal de agua.

66.- Masculino de 17 años, que presenta hemartrosis. Tiene TPT alargado, TP y TT


normales, por lo que tiene una alteración de la vía:

a) Del Complemento.
b) Intrínseca.
c) Extrínseca.
d) Colinérgica.

Manual CTO 7° edición, p. 723.

La vía intrínseca está constituida por la activación secuencial de los factores XII, XI, IX,.
VIII y V. el tiempo de tromboplastina parcial activada mide la actividad de la coagulación
intrínseca y sirve para monitorizar el tratamiento con la heparina no fraccionada.
67.- La principal causa etiológica para desarrollo de EPOC es:

a) Contaminación Ambiental.
b) Infecciones por Estreptococo.
c) Infecciones por Estafilococo.
d) Tabaquismo.

El tabaquismo es el factor de riesgo más importante para el desarrollo de EPOC; casi todos
los pacientes con EPOC sintomático son fumadores actuales o fueron, en los hombres que
fuman 20 cigarros al día la reducción en el FEV1 por año es de 9 ml más que en los no
fumadores.

Fishman, A; Manual de Enfermedades pulmonares, tercera edición, Mc Graw Hill, pags


130-131, 2004.

68. - The Virchow triad of thrombus in the etiology of pulmonary embolism is constituted
by:

a) Venous stasis, hypercoagulability, and endothelial damage.


b) Venous stasis, endothelial injury and hypoxemia.
c) Hypoxemia, hypercoagulability and cough.
d) Dry Cough.

Tromboembolismo pulmonar

Etiología:

El 90% de los casos de tromboembolismo pulmonar tienen su origen en venas de las


extremidades. Es favorecida por la triada de Virchow: éstasis venosa, daño de la íntima,
aumento de la coagulabilidad (puerperio, cirugía mayor, cáncer, uso de anticonceptivos,
policitemia vera, síndrome de hipercoagulabilidad (deficiencia de PC, PS, ATIII, resistencia
a la proteína C).

Fisiopatología.

Efectos respiratorios: aumento del espacio muerto alveolar, broncoconstricción,


taquipnea e hipoxemia, alteración ventilación/perfusión (V/Q) por redistribución de flujo.

Efectos hemodinámicos: La reducción mecánica leve o moderada no produce


aumento de la resistencia en forma significativa. Sobre el 50% de los casos presentan
incremento brusco de la resistencia y presión. Al efecto del émbolo se suman las aminas
liberadas por las plaquetas. En el 60-70% se desencadena por pulmonale agudo con
disminución brusca del gasto cardiaco.

Manifestaciones clínicas del tromboembolismo pulmonar:

Disnea, Taquipnea, Dolor pleurítico, Crepitantes, Tos, Taquicardia, Hemoptisis,


broncoespasmo, cianosis, síncope, palpitaciones. Síndromes: disnea aguda de causa
desconocida, hemoptisis y/o dolor pleurítico, shock cardiogénico.

British Thoracic Society, Standards of Care Committee. Suspected acute pulmonary


embolism: A practical approach. Thorax 1.997; 52 (suppl 4): S1-S23.

The PIOPED Investigators. Value of the ventilation/perfusion scan in acute


pulmonary embolism: results of the Prospective Investigation of Pulmonary
Embolism (PIOPED). JAMA 1.990; 263: 2.753-9.

Goodman PC. Spiral CT for pulmonary embolism Sem Resp Crit Care Med 2.000;
21(6): 503-10.

69.- Una mujer de 40 años de edad en buena salud general experimenta dolor retroesternal
súbito con fiebre y falta de aire. Es fumadora y no toma medicamentos excepto
anticonceptivos orales. En la exploración física se encuentran taquipnea y temperatura de
38°C. Los datos de auscultación, percusión y radiográficos del tórax son normales. ¿Cuál de
los siguientes diagnósticos es más probable?

a) Traqueobronquitis.
b) Neumonía atípica.
c) Embolia pulmonar.
d) Neumonía bacteriana.

Allen R. M. MMS Medicina Interna. 5ª. Edición. National Medical Series. Mc. Graw Hill.
2006. (capítulo 2 VIII E 1, 2 a; capítulo 8 V C 3, 4). El diagnóstico más probable es embolia
pulmonar. El inicio agudo descarta neumonía atípica y hace poco probable el cáncer
pulmonar. Sin pruebas de tos productiva es poco probable que haya traqueobronquitis,
trastorno también subagudo. La neumonía bacteriana es muy improbable junto con la
radiografía de tórax. El tabaquismo y el uso de anticonceptivos orales predisponen a
trombosis venosa profunda y embolias pulmonares.

Guadalajara J. Cardiología. Sexta Edición 151 – 152 La fibrilación auricular es la


arritmia cardiaca más frecuente. La despolarización caótica y desordenada de las aurículas
por múltiples ondas en simultánea, trae como consecuencia que se pierda la función
mecánica de la contracción auricular. Estos dipolos de activación múltiple y desordenados
alcanzan al nodo A-V y penetran en él, algunos pasan hacia el Haz de His mientras que otros
no lo alcanzan, debido a la penetración parcial del nodo por conducción decreciente
completa, esto se conoce como, conducción oculta; así, la rápida penetración de los
estímulos auriculares favorece la aparición de conducción oculta, la cual afecta en forma
impredecible el periodo refractario del nodo. El trazo electrocardiográfico típico es la
ausencia de onda P, complejo QRS normal e intervalos R-R diferentes.

70.- Una mujer de 25 años de edad presenta fiebre y artritis inflamatoria que afecta a las
articulaciones metacarpofalángicas e interfalángicas proximales ¿Cuál de las siguientes
manifestaciones es muy sugestiva de lupus eritematoso diseminado y no de artritis
reumatoide?

a) Sedimento urinario activo (eritrocitos, leucocitos, cilindros celulares, sin bacterias)


b) Artritis inflamatoria de articulaciones metacarpofalángicas e interfalángicas.
c) Derrame pleural de la radiografía de tórax.
d) Anemia.

Allen R. M. MMS Medicina Interna. 5ª. Edición. National Medical Series. Mc. Graw
Hill. 2006. (capítulo 10IIG;VIIG l;cuadro 10-10). Un sedimento urinario activo sugiere
glomerulonefritis, dato común en pacientes con lupus eritematoso diseminado, pero no en
quienes tienen artritis reumatoide. Pueden encontrarse artritis de articulaciones
metacarpofalángicas e interfalángicas proximales derrames pleurales y anemia en
cualquiera de las dos enfermedades. Las anomalías de la función hepática son atípicas en el
lupus eritematoso sistémico, pero comúnmente son resultado de disfunción hepática
relacionada con fármacos (p. ej., por antiinflamatorios no esteroideos en cualquier
enfermedad.

71.- Un varón de 73 años acude a una cena familiar de celebración y pasa una gran noche. Al
dejar el restaurante sufre un colapso. ¿Cuál de los siguientes trastornos es más probable?

a) Convulsiones.
b) Arritmia cardíaca.
C) Quiste coloide del tercer ventrículo.
d) Síncope posprandial.

Allen R. M. MMS Medicina Interna. 5ª. Edición. National Medical Series. Mc. Graw Hill.
2006. (capítulo 11IIA 3 a (2), b (2), d (1) (8)). El síncope posprandial es causa común de
desvanecimiento en ancianos. El consumo de alcohol también puede producir síncope y a
menudo es un factor contribuyente. Puede presentarse síncope posprandial cuando la
sangre es desviada al lecho mesentérico, lo que produce disminución relativa del riego
cerebral. El paciente no muestra datos de actividad convulsiva; por tanto, es poco probable
que se trate de una convulsión. Las arritmias cardíacas y la hipersensibilidad del seno
carotídeo pueden causar síncope y deben considerarse siempre en ancianos. Sin embargo, el
medio en que este paciente se desvaneció indica que es más probable un síncope posprandial.
Un quiste coloide del tercer ventrículo es causa rara de pérdida súbita de la conciencia. Los
quistes pequeños actúan como válvulas de bola y obstruyen el flujo por el agujero de
Monrow, lo que causa hidrocefalia aguda.

72.- Principal causa de Edema Pulmonar de tipo Hidrostático:

a) Neumonía.
b) Insuficiencia Cardiaca.
c) Bronquiectasias.
d) Bronquiolitis.

Un sistema de clasificación para distinguir el edema pulmonar es edema hidrostático y


edema por permeabilidad incrementada. El edema hidrostático como el que se ve en la
insuficiencia cardiaca es el resultado de un gradiente de presión incrementado en la pared
capilar.

Fishman, A; Manual de Enfermedades pulmonares, tercera edición, Mc Graw Hill,


Págs. 511-512, 2004.

73.- Anatomopatológicamente el asma bronquial se caracteriza por:

a) Estrechamiento de la luz bronquial y edema.


b) Estrechamiento de la luz bronquial y engrosamiento de la pared bronquial.
c) Edema y extravasación de líquidos.
d) Edema e Insuficiencia cardiaca.

La anatomía patológica del asma incluye el estrechamiento de la luz bronquial y


engrosamiento de la pared de la vía respiratoria. En la luz se observan moco y células
epiteliales así como infiltrado de células inflamatorias.
Fishman, A; Manual de Enfermedades pulmonares, tercera edición, Mc Graw Hill,
Págs. 129-130, 2004.
74.- Un cuadro de diarrea con una duración de más de dos semanas, pero que generalmente
no se extiende por más allá de cuatro se define como:

a) Diarrea aguda.
b) Diarrea persistente.
c) Diarrea crónica.
d) Diarrea acuosa.

La clasificación de la diarrea de acuerdo al tiempo de duración de este síntoma la divide en:


aguda, con duración de menos de 14 días, crónica con duración de más de 4 semanas, y
persistente ha la que tiene una duración menor del mes, y mayor a las dos semanas.

Kasper DL, Braunwald E, Fauci AS, Hauser SL, Longo DL, Jameson JL. Harrison´s
Principles of Internal Medicine. McGraw Hill. 16 Ed. 225 p.

75.- ¿Cuál es la osmolaridad plasmática normal?

a) 280 – 295 mOsm /Kg.


b) 295 – 300 mOsm /Kg.
c) 300 – 350 mOsm /Kg.
d) 400 – 500 mOsm /Kg.

La osmolaridad plasmática es la concentración molar de todas las partículas


osmóticamente activas en un litro de plasma.

Los valores normales son los siguientes:

- osmolalidad plasmática normal = 280 a 295 mOsm/kg

- osmolalidad urinaria = 50 a 1300 mOsm/kg

REGULACIÓN DE LA OSMOLARIDAD PLASMÁTICA

La osmolaridad plasmática normal se sitúa entre 275-290 mOsm/kg. Habitualmente se


mantiene dentro de estrechos límites, de modo que variaciones del 1-2% inician ya los
mecanismos compensadores. Estas alteraciones de la osmolaridad estimulan los
osmorreceptores hipotalámicos que influyen sobre la ingesta de agua mediante el
mecanismo de la sed y sobre su excreción renal por mediación de la ADH. La vasopresina
controla la osmolaridad entre 285 y 295 mOsm/kg, donde se encuentra el nivel máximo de
antidiuresis (por saturación de los receptores renales). Por ello, aunque el estímulo de la
sed se inicie desde los 285 mOsm/kg, es a partir de los 295 cuando es absolutamente
necesaria. Un exceso de agua (hipoosmolaridad) suprime la secreción de ADH, lo que
resulta en un descenso de la reabsorción tubular renal de agua y un aumento de la
excreción, retrasándose la diuresis máxima entre 90-120 minutos, el tiempo necesario para
metabolizarse la ADH circulante.

1. Adrogué, HJ. Madias, NE. Hyponatremia. N Eng J Med 2000; 342: 1581-1589.
2. Adrogué, HJ. Madias, NE. Hypernatremia. N Eng J Med 2000; 342: 1493-1499.
3. Rose BD, Post TW. Clinical physiology of acid-base and electrolyte disorders, 5th
edition. New York, Mc Graw-Hill 2001.

76.-Femenino de 44 con dolor epigástrico que la despierta por la mañana y se presenta


también cuando tiene hambre. El comer o el tomar antiácidos lo mejora. Se la realiza una
endoscopia que revela una úlcera duodenal. ¿Cuál es la etiología más probable de esta úlcera?

a) Uso de AINES.
b) Síndrome de Zollinger Ellison.
c) Infección por H. pylori.
d) Infección por C. Difficile.

La infección por Helicobacter pylori es un hallazgo frecuente en la población general,


siendo su prevalencia mayor en personas que padecen patologías gastrointestinales,
principalmente úlcera péptica (UP) y gastritis crónica (1).

Hasta 1983 la secreción ácida del estómago se consideraba el principal factor causal de la
úlcera duodenal, por ello el tratamiento de elección eran los fármacos antisecretores. Así,
a la cimetidina le siguieron otros antagonistas H2 (ranitidina, famotidina, nizatidina) y los
fármacos inhibidores de la bomba de protones (omeprazol, lansoprazol). Pero aunque estos
medicamentos supusieron un avance no lograron reducir el número de recidivas ulcerosas.
Este hecho supuso un estímulo para la búsqueda de alternativas.
Cuando en septiembre de 1983 Marshall (1) confirmó el hallazgo de H. pylori en las
muestras de biopsias gástricas de enfermos afectados de gastritis y úlceras duodenales,
se revolucionó el tratamiento de esta patología: las úlceras duodenales curan y cicatrizan
por completo con antisecretores, pero la infección por H. pylori permanece; por tanto, el
tratamiento más adecuado será aquel que consiga la eliminación duradera del
microorganismo (erradicación), ya que previene la recidiva de la úlcera.

1 . Marshall B J, Armstrong J A, McGechie D B y Glancy


R J. Attempt to fulfil koch’s postulates for pyloric campylobacter. Med J Aust 1985; 142:
436-9.
2. Taxonera C. Algunos aspectos del Helicobacter pylor
i en patología gastroduodenal. Tiempos Médicos 1994; 107-14.
3. Raws E A J. Role of Helicobacter pylori in duodenal ulcer. Drugs 1992; 44: 921-7.
4. Ateshkadi A, Lam N P y Johnson C A. Helicobacter pylori and peptic ulcer disease. Clin
Pharm 1993; 12: 34-48.
5. Hentschel E, Brandstatter G, Dragosics B, Hirschl A
M, Nemec H, Schutze K et al. Effect ranitidine and amoxicillin plus metronidazol on the
eradication of
Helicobacter pylori and the recurrence of duodenal ulcer. N Engl J Med 1993; 328: 308-
12.
6. Labenz J y Börsch G. Evidence for the esential role
of Helicobacter pylori in gastric ulcer disease. Gut 1994; 35: 19-22.
7. Graham D Y. Evolution of concepts regarding Helicobacter pylori: From a cause of
gastritis to a public
health problem (editorial). Am J Gastroenterol 199489: 469-72.
8. Courillon-Mallet A, Launay J M, Roucayrol A M,
Callebert J, Emond J P, Tabuteau F y Cattan D. Helicobacter pylori infection:
Physiophatologic implica -
tion of N-alfa-methyl histamine. G a s t r o e n t e r o l o g y 1995; 108: 959-66.
9. García Crávalos R y Pajares García.

77.- Se trata de femenino de 59 años, refiere vivir sola, como antecedentes refiere HTA
y artritis reumatoide, con buen control farmacológico. Presenta alucinaciones auditivas y
cenestésicas, con ideas delirantes de perjuicio con los vecinos y de contenido místico-
religioso de 4 meses de evolución. ¿Cuál sería el diagnóstico más probable?:

a) Síndrome confusional agudo.


b) Depresión delirante.
c) Esquizofrenia de inicio tardío.
d) Psicosis psicógena.

LOS ÚLTIMOS AÑOS se ha incrementado el interés por el estudio de los estados


psicóticos de inicio tardío (en mayores de 65 años) y su relación con el deterioro
cognoscitivo y los procesos demenciales ya instalados.
Las dificultades en la organización de un cuadro clínico específico de psicosis tardía que
responda a un estado nosológico, con características clínicas propias y a una etiología clara,
que le de cuerpo como una entidad, ha generado confusión entre los diversos investigadores
clínicos y epidemiológicos.

Esquizofrenia tardía: se caracteriza por la presencia de delirios, con alucinaciones y con


desorganización de la personalidad, alteración afectiva y conductas bizarras. Hay un menor
grado o ausencia de deterioro cognoscitivo, con una edad de inicio entre los 45 y 60 años.
No hay antecedentes de trastornos psicóticos, afectivos o demenciales.
. Esquizofrenia de inicio muy tardío en el anciano: son cuadros clínicos de tipo psicótico en
ancianos-ancianos (mayores de 60 años) que presentan sintomatología delirante poco
estructurada, con alucinaciones, con ausencia de deterioro cognoscitivo generalizado y
progresivo, sin compromiso afectivo significativo.
Existen otros estados psicóticos no esquizofrénicos de aparición en edades avanzadas,
como: el trastorno delirante de ideas persistentes, la paranoia, el trastorno delirante de
perjuicio, los delirios hipocondríacos de enfermedad y los delirios erotomaníacos.
Cuadro clínico
Diversos autores han intentado caracterizar un cuadro clínico de las psicosis de inicio
tardío en donde predominan los delirios, las alucinaciones, interpretaciones paranoides
(errores de percepción) y otros síntomas Scheneiderianos. En ancianos con psicosis tardías
el cuadro clínico puede estar conformado por:

Fenómenos delirantes, especialmente paranoides de tipo persecutorio o de referencia.


Pueden presentar un tipo especial de delirios denominados de tabique, los cuales se
caracterizan por la creencia de que detrás de las paredes de su habitación operan personas
con el propósito de hacerles daño o conspirar contra sus intereses e interferir contra su
vida.
. Fenómeno del eco del pensamiento: manifiestan que sus pensamientos son leídos o robados.
. Presencia de alucinaciones especialmente auditivas, con contenidos de referencia,
persecutorios o eróticos. También pueden presentar alucinaciones visuales, táctiles y
olfativas.
. Ausencia de trastornos cognoscitivos generalizados y progresivos.
. Los trastornos afectivos no son significativos, de tal modo que se sospeche de cuadros de
trastorno esquizoafectivo; pero estos ancianos psicóticos pueden presentar cuadros
depresivos moderados, sin que lleguen a constituirse en una depresión mayor de tipo
psicótico.
. Edad de inicio por encima de 60 años.
. Co-morbilidad frecuente con personalidad premórbida esquizoide o paranoide.
. Alteraciones del comportamiento se presentan más en el período de estado de la crisis
psicótica, pero en general hay menos desorganización y conductas bizarras que en aquellos
con cuadros crónicos de esquizofrenia.
. Falta de insight o introspección.
. Signos neurológicos blandos como reflejo glabelar, temblor, movimientos anormales,
discinesia tardía, hipoacusia, rigidez.ALARCÓN R.
Alteraciones Psiquiátricas en la Demencias, en Arango LJC, Fernández GS y Ardila A, Las
Demencias: Aspectos Clínicos, Neuropsicológicos y Tratamiento, Ed. Manual Moderno,
México,México, 2003
ALMEIDA OP, HOWARD R, LEVY R, DAVID AS. Psychotic states arising in late life (late
paraphrenia). The role of risk factors. Br J Psychiatry,
1995; 166: 215-228AMORES GF.
Funciones Cognitivas Superiores, Taller Master de Psicogeriatría, Universidad Autónoma de
Barcelona,
Barcelona, Febrero, 2004CASTLE D, MURRAY RM.
The epidemiology of late onset schizophrenia. Schizophr Bull, 1993; 19: 691-700
CHRISTENSON R, BLAZER DG.
Epidemiology of persecutory ideation in an elderly population in the community. Am J
Psychiatry,1984; 141: 1088-1091.
78.- Los gemelos unidos se forman cuando:

a) La división del ovulo fertilizado ocurre en la fase de mórula.


b) En el cuarto y octavo día post fecundación antes de la diferenciación de las células del
amnios.
c) La división ocurre después de la formación del disco embrionario.
d) La división ocurre cuando el amnios ya esta establecido ,8 días después de la
fertilización.

El embarazo gemelar siempre ha sido visto con mucho interés y, como es conocido, existen
dos tipos; los dicigóticos o fraternos, que representan el 70 %, y los monocigóticos, que son
alrededor del 30%.
En ocasiones los gemelos monocigóticos no se separan por completo y se producen los
llamados gemelos unidos o conectados, y se clasifican según la región anatómica que los une,
desde dos individuos bien desarrollados, independientes y simétricos, unidos tan sólo por
una pequeña conexión superficial, hasta los que están representados sólo por porciones de
cuerpo mutuamente adheridos o incluidos en un huésped más desarrollado
El proceso de desarrollo de los siameses ocurre por un error en la división de las células de
los embriones monocigóticos, es decir gemelos producto de un mismo óvulo y un mismo
espermatozoide. La
división de los embriones de unos gemelos normales ocurre normalmente en los primeros
diez días después de la fecundación, con los siameses esta división de embriones ocurre
más tarde, como para el día 13, este retraso hace que no se dividan correctamente y
compartan órganos. Mientras más tarde sea la separación más órganos compartirán.
Este no es un defecto genético sino que es una mutación durante la gametogénesis o el
desarrollo poscigótico. Por lo tanto, los padres y generaciones previas no presentan dicho
carácter. Por otra parte, hay ocasiones en donde estos gemelos siameses pueden ser
separados por medio de intervenciones quirúrgicas, dependiendo de la índole de la unión
entre los individuos, es decir si son simétricos o asimétricos
El origen no está aún bien definido, pero se señala que los gemelos siameses son el
resultado de una aberración en el proceso de formación de los gemelos monocigóticos, es
decir gemelos producto de un mismo óvulo y un mismo espermatozoide. De hecho, los
gemelos monocigóticos son en sí mismos considerados una aberración del desarrollo normal,
por tanto, los siameses representan un defecto aún más severo en el cual un gemelo
uniovular falla en su completa separación. La separación incompleta se considera como el
resultado de una duplicación parcial que ocurre en el disco embrionario antes de la tercera
semana de vida intrauterina.
. León J: Tratado de obstetricia. Buenos Aires: Ed Científica Argentina Rauch 1959(3):
229-46.
2. Potter EL: Pathology of the fetus and infant. 2ª ed. Chicago: Year Book Medical
Publishers Inc 1961.
3. Wilson H: Gemelos unidos. En: Benson C, Mustard W, Ravitch MM, Snyder WH, Welch
KJ: Cirugía infantil. La Habana: Edit Científico Técnica 1967; t1: 626-33.
4. Peñalver R: La desunión de las siamesas Maylín. Bohemia 1974; 22: 32-35.
5. Wallace I, Wallace A: Los siameses. La verdadera historia de los hermanos siameses.
Barcelona: Ed. Grijalbo, SA, 1978.
6. Votteler TP: Conjoined twins. En: Welch KJ et al: Pediaric surgery. Chicago: Year Book
Medical Publishers 1986; (t2): 771-9.
7. Sakala EP: Obstetric management of conioined twins. Obstet Gynecol 1986; 67(3 Suppl):
21S-25S.

79. Paciente de 52 años de edad con diagnóstico de neumonía adquirida en la comunidad,


quien recibió tratamiento antibiótico a base de Ceftriaxona. No tuvo respuesta adecuada, y
en el estudio diagnóstico para determinar la causa, se encontró un derrame pleural del 60%,
el cual se puncionó. En el estudio citoquímico y bacteriológico de este líquido, se reportan
cocos grampositivos en la tinción de Gram, y un pH de 7. Con esto, usted considera como
indispensable:

a) Iniciar cobertura con vancomicina.


b) Colocación de sonda endopleural.
c) Intubación orotraqueal y ventilación con volúmenes altos.
d) Ingreso a Unidad de Terapia Intensiva.

Los hallazgos del citoquímico y tinción son característicos de empiema. El drenaje del
mismo es la maniobra terapéutica base para el tratamiento y resolución del mismo.

Kasper DL, Braunwald E, Fauci AS, Hauser SL, Longo DL, Jameson JL. Harrison´s Principles
of Internal Medicine. McGraw Hill. 16 Ed. 1536 p.

80. Masculino de 65 años de edad, que se presenta porque desde hace 6 semanas presenta
disminución en el calibre de las heces, hematoquezia y constipación. Se realiza colon por
enema encontrando una zona de estenosis en el sigmoides, con imagen en manzana mordida.
Estos datos le hacen sospechar fuertemente en:

a) Enfermedad diverticular
b) Cáncer de colon
c) Apendicitis
e) Enfermedad de Crohn

Cotran R, Kumar V. Robbins Patología estructural y funcional. 6° edición. Mc Graw Hill.


Colombia 2003. 866-871. Pedrosa C, Casanova R. Diagnóstico por imagen. Mc Graw Hill 2001.
p 345, 346.

El cáncer de colon se presenta con: dolor abdominal mal definido, hematoquezia, anemia
asintomática, fatiga, pérdida de peso, cambios en el hábito intestinal, meteorismo doloroso,
disminución del calibre de las heces, constipación y obstrucción. La pérdida de peso no es
común. Desde el punto de vista radiológico pueden ser polipoideos, anulares, lesiones en
corazón de manzana o manzana mordida, obstructivas e incluso con ulceraciones. La más
frecuente es la lesión de tipo anular; el tumor crece desde su punto de origen, abarcando la
circunferencia del colon y produce un área de estrechez.

También podría gustarte